MSK SAEs Flashcards

1
Q

A 72-year-old woman underwent right total knee arthroplasty 2 days ago. When you see her in

consultation, she tells you that she has numbness along the lateral portion of the incision site.

What is the most likely cause?

(a) Femoral or peroneal nerve injury
(b) Deep vein thrombosis
(c) Cutaneous nerve injury
(d) Temporary side effect from anesthesia

A

Answer: (c)

Commentary: Cutaneous sensory loss is a very common complication following primary total

knee arthroplasty. One study from 1995 found that 100% of patients had lateral skin flap

numbness, and more recent studies in 2004 and 2009 found 81%-86% of patients had lateral skin

flap numbness.In most cases, the numbness does improve with time (50% recovered in 2 years in the 2009study). Deep vein thrombosis (DVT) and common peroneal nerve palsy are other known

complications of total knee arthroplasty.

How well did you know this?
1
Not at all
2
3
4
5
Perfectly
2
Q

Imaged with musculoskeletal ultrasound, normal tendon structure looks

a) hypoechoic, with hyperechoic septa.
b) hypoechoic , with fascicular pattern.
c) hyperechoic, with fibrillar echotexture.
d) hyperechoic, with posterior acoustic shadowing.

A

Answer:(c)

Commentary: Musculoskeletal ultrasound is an imaging modality that is able to identify and

characterize various soft tissue structures. Normal tendons appear as hyperechoic(bright echo)

structures with fibrillar or fiber-like pattern. Normal muscle appears as a hypoechoic(low echo)

structure with hyperechoic septa. Bone appears as a very hyperechoic structure with posterior

acoustic shadowing. Posterior acoustic shadowing is an artifact that refers to the anechoic

region(no echo) deep to the bone surface.

How well did you know this?
1
Not at all
2
3
4
5
Perfectly
3
Q

The third occipital nerve innervates which structure?

(a) C2-3 zygapophysial joint
(b) C2-3 intervertebral disc
(c) C3-4 zygapophysial joint
(d) C3-4 intervertebral disc

A

Answer:(a)

Commentary: The third occipital nerve(TON) innervates the C2-3 zygapophysial joint. The C3-4

zyagpophysial joint is innervated by the C3 and C4 medial branches. Innervation to the cervical

discs involves the sinuvertebral nerve, vertebral nerve and sympathetic trunk.

How well did you know this?
1
Not at all
2
3
4
5
Perfectly
4
Q

SAER-2011

  1. Injured workers with acute low back pain treated with high-dosage opioids compared to low-dose

or nonopioid medications demonstrated which outcome?

(a) Lower overall medical costs
(b) Same duration of disability
(c) Higher risk for surgery
(d) Shorter duration of opioid use

A

Answer: (c)

Commentary: Injured workers with acute low back pain who received higher dosages of opioids

in early treatment had adverse outcomes compared to patients given no or low-dose opioids. In

the high-dose opioid group, adverse outcomes included higher medical costs, prolonged

disability, higher risk for surgery, and continued use of opioids. The high-dose opioid group was

disabled 69 days longer than the non-opioid group, had a 3 times greater risk for surgery, and a 6-

times-greater risk of receiving long-term opioids. The severity of the low back injury was a strong

predictor of all outcomes.

How well did you know this?
1
Not at all
2
3
4
5
Perfectly
5
Q
  1. Hamstring injuries occur most commonly
    a) at the proximal attachment of the lateral hamstrings to the pelvis.
    b) during concentric contraction of the medial hamstrings.
    c) at the distal attachment of the medial hamstrings to the tibia.
    d) during eccentric contraction of the lateral hamstrings.
A

Answer: (d)

Commentary: The majority of hamstring injuries occur from indirect forces during running and

sprinting activities. Most injuries occur at the myotendinous junction, not at the osseous

attachments, during eccentric contraction of the hamstring. The lateral hamstrings (biceps

femoris) are affected more than the medial hamstrings (semitendinous and semimembranosus).

How well did you know this?
1
Not at all
2
3
4
5
Perfectly
6
Q
  1. For injured workers with chronic low back pain, which outcome is associated with better

performance during a functional capacity evaluation (FCE)?

(a) Shorter usage of temporary disability benefits
(b) Lower subjective reports of perceived disability
(c) Higher likelihood of sustainable work tolerance
(d) Fewer recurrences of low back pain over the next 12 months

A

Answer: (a)

Commentary: Functional capacity evaluations (FCEs) are commonly used to determine readiness

for return to work. These evaluations measure the injured worker’s functional abilities relative to

the physical demands required by the job. The clinical assumption is that workers who perform

better during FCEs will have a lower risk of reinjures and less pain exacerbation upon return to

work. One-year follow-up of patients with chronic low back pain whose FCE demonstrated

performance that met or exceeded physical job requirements did not demonstrate a reduction of

recurrent low back pain, improved occupational sustainability, or improved perception of

disability. Better FCE performance was mildly associated with faster return to work and shorter

duration of temporary disability benefits.

How well did you know this?
1
Not at all
2
3
4
5
Perfectly
7
Q
  1. Which statement regarding an independent medical examination (IME) is TRUE?
    (a) The traditional physician-patient relationship is not maintained, and confidentiality is not
    guaranteed.
    (b) The examiner is exempt from potential liability since the purpose of the evaluation is to

assess medical-legal issues, not clinical issues.

(c) Treating providers may conduct an IME as long as records from other providers are also
reviewed.
(d) Because of potential conflicts of interest, only providers no longer in clinical practice

should conduct IMEs.

A

Answer: (a)

Commentary: In the IME context, a traditional physician-patient relationship does not exist, since

the evaluation does not include “intent to treat.” Confidentiality is not guaranteed, since the

examiner is expected to share certain medical information and findings with the referring party.

Because a “limited doctor-patient relationship” exists during an IME, the physician is responsible

for disclosing in the IME any medical findings that could affect the patient’s health, and he or she

is potentially liable for any harm, direct or indirect, that may be sustained by the person

examined. Only a provider who is uninvolved with an examinee’s treatment may conduct an

IME, although a treating provider may be an “expert witness.” Legal requirements for

qualification as an expert witness vary from state to state. There is no restriction regarding a

provider’s clinical status and eligibility to conduct IMEs.

How well did you know this?
1
Not at all
2
3
4
5
Perfectly
8
Q
  1. Comparing the functional outcomes at 1-year post treatment of 2 groups of patients with

nonspecific low back pain greater than 12-months’ duration and no prior history of lumbar fusion,

which finding regarding structured rehabilitation with cognitive behavioral therapy (CBT) versus

lumbar fusion is TRUE?

(a) Better functional outcomes in the surgical group versus the CBT group
(b) Improvements in both groups with similar functional outcomes
(c) Better functional outcomes in the CBT group versus the surgical group
(d) Poor functional outcomes in the CBT group, but no consistent outcome in the surgical Group

A

Answer: (b)

Commentary: Randomized trials for surgery are difficult to conduct, particularly those that

compare surgical to nonsurgical treatment. While available studies do not allow a general

statement regarding the efficacy of fusion over nonsurgical care for discogenic back pain, 4 trials

suggest any advantage of surgery over nonsurgical care is modest, on average near or below the

minimally important change in the disability score. Both groups demonstrated improvement

compared to baseline. Highly structured rehabilitation with a cognitive-behavioral component

seems nearly equivalent to surgery in efficacy at 1 year, with fewer complications.

How well did you know this?
1
Not at all
2
3
4
5
Perfectly
9
Q
  1. A firefighter who is now 5 days postsurgery for a rotator cuff and labral tear is in significant pain, but is concerned about opioid use for pain control. He is concerned about becoming “addicted to the pain killers.” In educating the patient about opioids and the issues of addiction, dependence and tolerance, which statement is correct?
    (a) While all 3 terms have subtle differences, they are essentially identical in meaning and

can be used interchangeably.

(b) Since he is a firefighter, he should avoid use of any opioids at all times since he is subject

to toxicology screening.

(c) Addiction is predictable and avoidable, and since he already concerned about it, he is

unlikely to have problems with addiction.

(d) Addiction is characterized by behavioral issues, whereas dependence and tolerance are

characterized by physiologic adaptation.

A

Answer: (d)

Commentary: Physical dependence, tolerance, and addiction are discrete and different phenomena

that are often confused. Addiction is characterized by behaviors that include one or more of the

following: impaired control over drug use, compulsive use, continued use despite harm, and
craving. Addiction is not a predictable drug effect, but represents an idiosyncratic adverse

reaction in biologically and psychosocially vulnerable individuals. Physical dependence is a state

of adaptation characterized by specific withdrawal symptoms that can be produced by abrupt

cessation, rapid dose reduction, and/or administration of an antagonist. Tolerance is a state of

adaptation that results in a decreased effect of a drug over time.

How well did you know this?
1
Not at all
2
3
4
5
Perfectly
10
Q
  1. You are asked to provide a brief synopsis of workers’ compensation benefits to the hospital’s case management department. Which statement about the benefits and services provided in the

workers’ compensation system is correct?

(a) The employer has to be at fault in order for the injured employee to seek medical care.
(b) Injured workers continue to receive their full wages as long as they are unable to work.
(c) Workers’ compensation programs are designed and administered by each individual state.
(d) Workers’ compensation is primarily financed by federal and state funds.

A

Answer: (c)

Commentary: Workers’ compensation provides benefits to workers who are injured on the job or have a work-related illness, regardless of who is at fault for the injury or illness. Benefits include medical treatment for work-related conditions and cash payments that partially replace lost wages. In the event that symptoms do not completely resolve, financial compensation is also provided. In exchange, an injured worker gives up the right to sue the employer because of a work-related injury or illness. Workers’ compensation programs are designed and administered by each state, and programs and policies vary from state to state. Workers’ compensation is financed almost exclusively by employers, not federal or state funds.

How well did you know this?
1
Not at all
2
3
4
5
Perfectly
11
Q
  1. A 22-year-old man who is right hand-dominant presents to your office with acute onset of right shoulder pain. He is a former college tennis player without a previous history of shoulder
    pathology. Which test would you perform to evaluate for pathology involving the labrum of the

shoulder?

a) Hawkin test
b) Bowstring sign
c) O’Brien active compression test
d) Apley scratch test

A

Answer: (c)

Commentary: The O’Brien active compression test is used to evaluate and differentiate labral

tears (superior labrum anterior posterior – SLAP tears) from acromioclavicular joint pathology

and pain. The Hawkin test is an impingement test of the shoulder and is not intended to evaluate

tears of glenoid labrum. The bowstring sign is used to identify lumbar nerve root compression.

The Apley scratch test is used to assess the range of motion of the shoulder.

How well did you know this?
1
Not at all
2
3
4
5
Perfectly
12
Q
  1. A 30-year-old woman began running 2 weeks ago. She runs 4 miles a day, twice a week. She began experiencing bilateral lower leg pain. On physical examination, she reports diffuse pain along the medial tibia at the start of her run with improvement during the run. The most likely diagnosis is
    a) stress fracture.
    b) medial tibial stress syndrome.
    c) anterior tibialis tendinitis.
    d) tarsal tunnel syndrome.
A

Answer: (b)

Commentary: This woman presents with symptoms most consistent with medial tibial stress

syndrome (MTSS) or what has been termed “shin splints.” Pain from MTSS occurs along the

lower third of the posteromedial border of the tibia. A stress fracture is unlikely in this low

mileage runner who has had only 2 weeks of running activity. Stress fractures generally have a

focal area of pain and are not relieved with further running. Anterior tibialis tendinitis presents

with anterolateral pain along the dorsal aspect of the ankle. Tarsal tunnel syndrome is associated

with numbness and tingling in the foot.

How well did you know this?
1
Not at all
2
3
4
5
Perfectly
13
Q
  1. A man presents to your clinic complaining of buttock pain that radiates posteriorly down the thigh. On exam you note that he has a leg length discrepancy, symptoms are provoked by placing the affected limb in the FAIR position (hip in flexion, adduction and internal rotation), and he has a positive straight leg raise test. He has normal nerve conduction studies and a normal needle electromyography test. Treatments that may be beneficial include
    (a) stretching exercises of the iliotibial band and corticosteroid injection of the greater trochanteric bursae.
    (b) a lumbar stabilization exercise program and coricosteroid lumbar epidural spinal injection.
    (c) a lumbar stabilization exercise program and botulinum toxin injection of the lumbar paraspinals.
    (d) stretching exercises in the FAIR position and botulinum toxin injection to the piriformis.
A

Answer (d)

Commentary: This is a description of piriformis syndrome. Although some positive findings on

needle examination may be seen with piriformis syndrome, electrodiagnostic studies are often

normal. On the other hand, positive findings are expected in cases of lumbar radiculopathy.

Conservative treatment of piriformis syndrome begins with piriformis stretching (FAIR position

is a good position for this) and nonsteroidal anti-inflammatory drugs (NSAIDs), followed by

lumbosacral stabilization, hip strengthening, and myofascial release. Botulinum toxin relieves

pain via multiple mechanisms and is increasingly used in the treatment of myofascial dysfunction.

A lumbar stabilization exercise program and botulinum toxin injection of the lumbar paraspinals

may help relieve some of this patient’s pain if he also has low back pain, but would not address

the main issue, piriformis syndrome. Stretching exercises of the iliotibial band and corticosteroid

injection of the greater trochanteric bursae would be the treatment for greater trochanteric

bursitis. A lumbar stabilization exercise program and corticosteroid lumbar epidural spinal

injection would treat a lumbar radiculopathy.

How well did you know this?
1
Not at all
2
3
4
5
Perfectly
14
Q
  1. 20-year-old football player reports anterior shoulder pain during a game. He completes the game, but radiographs after the game revealed a type 2 acromioclavicular (AC) joint sprain. How is a type 2 acromioclavicular (AC) joint injury defined?
    (a) Acromioclavicular and coracoclavicular ligaments are both disrupted.
    (b) Acromioclavicular and coracoclavicular ligaments are both intact.
    (c) Acromioclavicular ligament is disrupted, but the coracoclavicular ligament is intact.
    (d) Acromioclavicular ligament is intact, and the coracoclavicular ligament is disrupted.
A

Answer: (c)

Commentary: Acromioclavicular joint injuries are classified into 6 types according Rockwood

classification. A type 1 injury describes a mild injury to the AC joint without disruption of either

the acromioclavicular or the coracoclavicular ligaments. A type 2 injury describes disruption of

the acromioclavicular ligament, but the coracoclavicular ligament remains intact. A type 3 injury

describes disruption of both ligaments whereas a type 4 injury entails complete disruption of both

ligaments with posterior displacement of the distal clavicle into the trapezius muscle.

How well did you know this?
1
Not at all
2
3
4
5
Perfectly
15
Q
  1. Which statement is TRUE regarding the rehabilitation of anterior cruciate ligament (ACL)

reconstruction/repair?

(a) Immediate postoperative weight bearing adversely affects subsequent knee function.
(b) A self-directed program is not as effective as regular physical therapy visits.
(c) Use of a continuous passive motion machine improves outcome.
(d) Postoperative functional bracing does not improve outcome.

A

Answer: (d)

Commentary: The use of postoperative functional bracing does not improve outcome. Immediate postoperative weight bearing does not adversely affect subsequent knee function. A self-directed program is as effective as regular physical therapy visits in a motivated patient. The use of a continuous passive motion machine does not improve outcome.

How well did you know this?
1
Not at all
2
3
4
5
Perfectly
16
Q
  1. Compared to a younger individual, an older worker who suffers a musculoskeletal injury is more likely to
    (a) return to work sooner.
    (b) have a recurrent injury.
    (c) have the injury treated nonsurgically.
    (d) sustain a less serious injury.
A

Answer: (b)

Commentary: Compared to a younger individual who suffers a musculoskeletal injury, an older

individual is more likely to have a recurrent injury, a decreased likelihood of returning to work

after the injury, increased time lost from the job as a result of the injury and a more serious injury.

Also, an older individual with a spine injury is more likely to have surgery than is a younger

individual.

How well did you know this?
1
Not at all
2
3
4
5
Perfectly
17
Q
  1. A 40-year-old woman reports left-sided facial pain for the past month along with difficulty in

moving her jaw. She hears a clicking noise with chewing along with constant tinnitus. Upon

examination, she has tenderness to palpation along her muscles of mastication on the left with

deviation of the mandible upon jaw opening. She would like to have pain relief. You suggest

(a) referral to an oral surgeon.
(b) that she perform jaw isometric exercises in a closed position with massage.
(c) a 2-week trial of an oral nonsteroidal anti-inflammatory medication.
(d) an ultrasound-guided intra-articular injection with steroids.

A

Answer: (c)

Commentary: This woman has a temporomandibular joint (TMJ) disorder most likely myofascial

in origin, which is the most common etiology. It is usually self-limited, and is managed

conservatively with relative rest (eg, avoiding jaw clenching, gum chewing), heat, and

nonsteroidal anti-inflammatory agents. Intra-articular steroid injections are not needed with a

myofascial origin of pain. She also does not need a referral to an oral surgeon at this time.

How well did you know this?
1
Not at all
2
3
4
5
Perfectly
18
Q
  1. The current workers’ compensation system in the United States is a “no fault” system. This

means that the

(a) employee does not have to prove that the employer is at fault for the injury.
(b) employer does not have to prove they are at fault for the employee’s injury.
(c) employee and employer do not have to prove that the other is at fault for the injury.
(d) employer does not have to prove that the employee is at fault for the employee’s injury.

A

Answer: (c)

Commentary: In the United States workers’ compensation system the injured worker does not

have to prove that the employer is at fault for the employee’s injury. Similarly, the employer

does not have to prove that the injured worker is at fault for his/her injury. If the injury occurred

at work, the medical costs and partial payment of lost income are covered.

How well did you know this?
1
Not at all
2
3
4
5
Perfectly
19
Q
  1. An 87-year-old man on your inpatient rehabilitation unit was found on the therapy mat in much pain after hearing a loud “cracking” noise when he transferred himself. What position of his right lower limb would suggest hip fracture?
    (a) Internal rotation and lengthened
    (b) Internal rotation and shortened
    (c) External rotation and lengthened
    (d) External rotation and shortened
A

Answer: (d)

Commentary: In most cases, the lower limb of the fractured hip would be held in external rotation

(rotated outward) and would appear shortened relative to the unaffected lower limb.

How well did you know this?
1
Not at all
2
3
4
5
Perfectly
20
Q
  1. You are seeing a 79-year-old gentleman with chronic right shoulder pain. For the past several years he has had limited shoulder movement and is diffusely tender around the shoulder. Magnetic resonance imaging demonstrates a partial tear of the supraspinatus and infraspinatus tendons with degenerative changes of the glenohumeral joint. You recommend
    (a) rotator cuff repair.
    (b) total shoulder arthroplasty.
    (c) intra-articular viscosupplementation injection.
    (d) flexibility and progressive strengthening exercises.
A

Answer: (d)

Commentary: The nonsurgical management of shoulder osteoarthritis (OA) with a chronic,

massive rotator cuff defect requires flexibility exercises and gentle progressive strengthening

exercises to increase shoulder function. Surgical repair involves humeral hemiarthroplasty.

Rotator cuff repair in partial thickness tears consists of surgical smoothing of the humeroscapular

motion interface with cuff curettage. Reverse total shoulder arthroplasty is used for

anterosuperior escape rotator cuff lesions. There is no role for shoulder viscosupplementation,

since it has not been shown to be beneficial.

How well did you know this?
1
Not at all
2
3
4
5
Perfectly
21
Q
  1. Which clinical scenario is most consistent with a L4-5 foraminal disc herniation?
    (a) Weakness of the extensor hallicus longus, decreased sensation of the web space between

the first and second toes, absent hamstring reflex

(b) Weakness of the gastrocnemius, decreased sensation of lateral foot, absent Achilles reflex
(c) Weakness of the anterior tibialis, decreased sensation of the web space between the first

and second toes, absent hamstring reflex

(d) Weakness of quadriceps and anterior tibialis, decreased sensation of medial lower leg,

absent patellar reflex

A

Answer: (d)

Commentary: A foraminal disc herniation at L4-5 level would most likely affect the exiting L4

nerve root. A nerve root lesion could result in muscle weakness in the affected myotomes,

sensation loss in the affected dermatomes, and deep tendon reflex changes. The physical

examination findings most consistent with a lesion to the L4 nerve root would be weakness of

the quadriceps (L2-4), decreased sensation in L4 dermatomes, and decreased or absent patellar

deep tendon reflex (L4).

How well did you know this?
1
Not at all
2
3
4
5
Perfectly
22
Q
  1. A 67-year-old man presents to your clinic with weakness and frequent falls. You suspect cervical stenosis. Calculating anteroposterior (AP) ratios to other anatomical structures, which ratio would enable you to assess for bony cervical spinal stenosis on lateral radiographs?
    (a) AP diameter of the vertebral body to the height of vertebral body.
    (b) AP diameter of the vertebral canal to the AP diameter of the vertebral body at the same level.
    (c) Vertebral height to the AP of the vertebral canal at the same level.
    (d) Distance from the anterior border of the vertebral body to the tip of the spinous process.
A

Answer: B

Commentary: Assessment of cervical spinal stenosis on lateral radiograph can be made by calculating the ratio of the anteroposterior (AP) diameter of the vertebral canal to the AP diameter of the vertebral body at the same level. This ratio is called the Pavlov ratio. A normal ratio is 1.0 with less than 0.82 indicating stenosis. The Torg ratio is the same as Pavlov ratio.

How well did you know this?
1
Not at all
2
3
4
5
Perfectly
23
Q
  1. A 47-year-old woman injures her back on the job. Her supervisor inquires about the injury and creates a document with the employee’s name, outlining how the injury occurred and where the employee is experiencing pain. The information in the document is protected by the
    (a) Health Insurance Portability and Accountability Act (HIPAA).
    (b) The Joint Commission (JC).
    (c) Americans with Disabilities Act (ADA).
    (d) United States Supreme Court.
A

Answer: A

Commentary: Once the document was created by the supervisor and it contained individually identifiable health information it became information that is protected by HIPAA. The term ‘individually identifiable health information’ means any information, including demographic information collected from an individual, that: (A) is created or received by a health care provider, health plan, employer, or health care clearinghouse; and (B) relates to the past, present, or future physical or mental health or condition of an individual, the provision of health care to an individual, or the past, present, or future payment for the provision of health care to an individual, and identifies the individual.

How well did you know this?
1
Not at all
2
3
4
5
Perfectly
24
Q
  1. Repeatedly lifting the shoulder past which degree of flexion or abduction is associated with an increased prevalence of shoulder disorders?
    (a) 10o
    (b) 30o
    (c) 45o
    (d) 60o
A

Answer: D

Commentary: Repeatedly lifting the shoulder past 60 degrees of flexion or abduction is associated with an increased prevalence of shoulder disorders.

How well did you know this?
1
Not at all
2
3
4
5
Perfectly
25
Q
  1. Which factor is associated with increased risk for occupational injury in an older individual?
    (a) White collar occupation
    (b) Female gender
    (c) Impaired hearing
    (d) Self employment
A

Answer: C

Commentary: Predictors of increased injury risk in an older worker include male gender, less education, obesity, alcohol abuse, disability, self report of impaired hearing or sight, and several specific job requirements. Service workers, mechanics, machine operators, and laborers are at increased risk for occupational injury compared to people in white collar occupations. Individuals who are self-employed have a lower risk of injury.

How well did you know this?
1
Not at all
2
3
4
5
Perfectly
26
Q

A 35-year-old man presents to your clinic with a 3-month history of groin pain exacerbated by activity. He is an avid skier and runner. He has been taking anti-inflammatories with minimal relief. Anteroposterior films of the hip were normal. The magnetic resonance imaging of the hip reported a bony prominence at the femoral head-neck junction. What clinical exam finding is most likely to correlate with these radiographic “abnormalities”?

(a) Pain with resisted straight leg raise
(b) Pain with hip flexion, external rotation, and abduction
(c) Pain with sacral thrust
(d) Pain with hip flexion, internal rotation, and adduction

A

Answer:D

Commentary: This patient has radiographic evidence of femoroacetabular impingement. Two types have been described, cam impingement and pincer impingement. Cam impingement is described more commonly in active males and describes a non-spherical femoral head or osseous abnormalities of the femoral head-neck junction. These bony abnormalities have abnormal contact with the acetabulum in hip flexion, adduction, and internal rotation. Pincer impingement describes abnormal contact between the femur and the acetabulum due to overcoverage of the femoral head from an abnormally deep or retroverted acetabulum.

How well did you know this?
1
Not at all
2
3
4
5
Perfectly
27
Q
  1. A recent study of individuals undergoing a single knee or hip replacement surgery who were treated at an inpatient rehabilitation facility (IRF) compared to those treated at a skilled nursing facility (SNF) found that those treated in an IRF were more likely to
    (a) need the use of a walker to ambulate.
    (b) require home care services.
    (c) be discharged home.
    (d) ambulate a shorter distance.
A

Answer: C

Commentary: Patients undergoing single knee or hip replacement surgery who were treated at an IRF were more likely to be discharged home, less likely to require home care services upon discharge, and were able to ambulate farther distances compared to those treated in an SNF.

How well did you know this?
1
Not at all
2
3
4
5
Perfectly
28
Q
  1. A 55-year-old woman presents to the clinic with a 6-week history of right wrist pain. She is an administrative assistant and has been working extra hours for the past 3 months. She has been taking anti-inflammatory medications without relief. You diagnose her with de Quervain tenosynovitis. What is the next most appropriate step in treatment?
    (a) Trial of a higher dose of anti-inflammatory medication
    (b) Surgical consultation
    (c) Corticosteroid injection
    (d) Splinting the wrist
A

Answer: C

Commentary: Corticosteroid injection for de Quervain tenosynovitis has been shown to be more effective treatment than splinting and anti-inflammatory medications.

How well did you know this?
1
Not at all
2
3
4
5
Perfectly
29
Q
  1. Which structure is required by the American with Disabilities Act (ADA) to have adequate accessibility for individuals with disabilities, so long as the modifications to it are readily achievable?
    (a) House of worship
    (b) Physician’s office within a private residence
    (c) Commercial airplane
    (d) Residential private apartments
A

Answer: B

Commentary: If a publicly accessible office is present within a single family home, it is required to be accessible under the ADA, so long as the necessary modifications are readily achievable. Accessibility of commercial airplanes is covered under the Air Carrier Access Act, not the ADA.

Access to houses of worship or strictly residential private apartments is not required under the ADA.

How well did you know this?
1
Not at all
2
3
4
5
Perfectly
30
Q

22-year-old female volleyball player fell on an outstretched right hand 3 weeks ago and complains of continued wrist pain. On examination, she has minimal swelling of the distal limb and is tender to palpation distal to the ulnar styloid between the flexor carpi ulnaris and extensor carpi ulnaris tendons. A plain radiograph was normal except for an ulnar plus variant. She failed conservative treatment with splinting and activity modification. The most appropriate imaging study to obtain would be

(a) computed tomography scan of the wrist.
(b) repeat plain films in 10 days.
(c) triple phase bone scan.
(d) magnetic resonance imaging with arthrogram.

A

Answer: D

Commentary: This patient sustained an injury to her triangular fibrocartilage complex. This structure is a stabilizer of the distal radioulnar joint and is composed of an avascular articular disc and radioulnar ligament complex. It is often injured with repetitive wrist activities or compressive loads. Tears to it are best imaged by MRI arthrogram. Injury to this complex would not be optimally evaluated on plain films, bone scan, or computed tomography scan.

How well did you know this?
1
Not at all
2
3
4
5
Perfectly
31
Q
  1. You recommend work hardening for a worker recovering from a shoulder injury. You explain to the worker to expect a therapy program that
    (a) builds aerobic conditioning and will be performed 2 hours daily.
    (b) simulates work duties and will be performed approximately 4 hours daily.
    (c) simulates a heavy manual labor job and will be performed 6 hours daily.
    (d) improves aerobic conditioning while simulating a light duty job and will be performed 8 hours daily.
A

Answer: B

Commentary: Work hardening is a rehabilitation program designed to simulate the individual worker’s job. It can be performed at a center or at the worker’s jobsite. These programs are often recommended to be done 5 days a week. The worker performs an individualized program based

on his/her specific job requirements. Physician follow-up is needed to determine if goals have been achieved. Work conditioning is a program used to enhance aerobic conditioning but does not attempt to replicate the tasks of a specific job.

How well did you know this?
1
Not at all
2
3
4
5
Perfectly
32
Q
  1. A 60-year-old man with left total knee arthroplasty 5 days prior continues to have difficulty with ambulation during rehabilitation. On exam, he has 70oof active knee flexion, a 20oextensor lag, and a distal lower limb normal to palpation. You then notice that he has trouble clearing his toes during swing phase. You suspect the major cause of his difficulty walking is due to
    (a) weak quadriceps strength.
    (b) inadequate knee flexion range.
    (c) commonperoneal nerve palsy.
    (d) tibialis anterior tendon tear.
A

Answer: C

Commentary: The patient has a common peroneal nerve palsy which can occur after total knee arthroplasty. Weak quadriceps strength and inadequate knee flexion may cause difficulty with ambulation, but not the loss of ankle dorsiflexion. Tibialis anterior tendon tear will cause difficulty with ankle dorsiflexion, but is not a common complication after knee arthroplasty. Also, acute tendon tears present with sudden pain and palpatory defect.

How well did you know this?
1
Not at all
2
3
4
5
Perfectly
33
Q
  1. A 28-year-old man returns to clinic after failing conservative management for clinical medial epicondylitis. In order to determine the appropriateness of a surgical referral, what is the most cost effective diagnostic test to localize the site of pathology?
    (a) Plain radiographs of the elbow and forearm
    (b) Magnetic resonance imaging of upper extremity
    (c) Real time ultrasound
    (d) Electrodiagnostic studies
A

nswer: C

Commentary: Real time ultrasound is less costly than magnetic resonance imaging (MRI) and has similar sensitivity and specificity in diagnosing medial epicondylitis. Plain radiographs and electrodiagnostic studies will not help localize or confirm your diagnosis of medial epicondylitis, but may help with diagnosing a fracture or nerve injury, respectively.

How well did you know this?
1
Not at all
2
3
4
5
Perfectly
34
Q
  1. Disability as defined by the Americans with Disabilities Act (ADA) is
    (a) a physical or mental impairment that substantially limits 1 or more major life activities.
    (b) abnormality of the physiologic or anatomic structure or function.
    (c) the barriers society places on the individual interacting in his/her community.
    (d) a rating based on an independent medical examination.
A

Answer: A

Commentary: The Americans with Disabilities Act defines disability as a physical or mental impairment that substantially limits 1 or more of a person’s major life activities. The person has a record of such impairment, or is regarded as having such impairment. Impairment is the actual physiologic, anatomic, or psychologic abnormality. Handicap refers to the barriers society places on an individual to perform function in the community. A permanent disability rating is used to determine financial compensation for an injury.

How well did you know this?
1
Not at all
2
3
4
5
Perfectly
35
Q
  1. The validity of a functional outcome measurement tool is defined as the ability
    (a) of two different raters to obtain the same conclusion.
    (b) of the tool to measure what it is designed to measure.
    (c) to minimize random error.
    (d) to measure several different outcomes simultaneously
A

Answer: B

Commentary: The validity of a functional outcome measurement tool is defined as the ability of the tool to measure what it is designed to measure. The ability to measure different outcomes simultaneously does not impact the validity of the instrument, but the validity of the tool would need to be established for each of the outcomes being measured. The ability of two different raters to obtain the same conclusion is referred to as inter-rater reliability. Freedom from random error is also related to the reliability of the instrument.

How well did you know this?
1
Not at all
2
3
4
5
Perfectly
36
Q

. A 23-year-old postgraduate student presents to your office with bilateral knee pain. She just began training for a half marathon but has been limited by her knee pain. She reports pain in the anterior aspect of the knee and describes it as “beneath the knee cap.” The pain is worse when arising after sitting for a prolonged period of time. Which physical examination finding might you expect in this patient?

(a) Pes cavus
(b) Strong hip abductors
(c) Negative Ober test
(d) Tight quadriceps muscles

A

Answer: D

Commentary: Patellofemoral arthralgia is thought to result from tracking problems of the patella within the trochlear groove. Several biomechanical issues, such as tight and inflexible quadriceps, pes planus, tight iliotibial band, weak and ineffective vastalis medialis, and weak hip abductors, may contribute to incorrect tracking of the patella. The Ober test assesses the tensor fascia lata and iliotibial band for contracture and inflexibility.

How well did you know this?
1
Not at all
2
3
4
5
Perfectly
37
Q

. According to the American Medical Association Code of Ethics Opinion, which statement is TRUE?

(a) Individual gifts of minimal value from pharmaceutical representatives to physicians are permissible so long as the gifts are related to the physician’s work.
(b) It is acceptable for physicians to request free pharmaceuticals for personal use or use by family members.
(c) Subsidies to underwrite the costs of continuing medical education are permissible when provided directly from the pharmaceutical company to the physician.
(d) Faculty presenting at conferences cannot accept honoraria and reimbursement for travel, lodging, and meal expenses.

A

Answer: A

Commentary: According to the AMA Code of Ethics Opinion, individual gifts of minimal value from pharmaceutical representatives to physicians are permissible, so long as the gifts are related to the physician’s work. It is not acceptable for physicians to request free pharmaceuticals for personal use or use by family members. Subsidies to underwrite the costs of continuing medical education are permissible when they are accepted by the conference’s sponsor and are not provided directly to the physician. It is acceptable for faculty at conferences to accept reasonable honoraria and reimbursement for travel, lodging, and meal expenses.

How well did you know this?
1
Not at all
2
3
4
5
Perfectly
38
Q
  1. A 50-year-old administrative assistant presents with low back pain. After taking her history, performing a physical examination, and reviewing her imaging studies, you determine that her pain is likely discogenic. She asks if there are any positions which would be better for her back while at work. Which position exerts the most pressure on the lumbar discs?
    (a) Standing erect
    (b) Standing erect and flexed forward
    (c) Seated in a chair
    (d) Seated in a chair and flexed forward
A

Answer: D

Commentary: Nachemson measured the relative pressure changes within the third lumbar disc with changes of position. Standing erect was the reference position and pressures decreased with lying supine and increased in the seated position. Seated and flexed forward further increased disc pressures. Several other positions were evaluated.

How well did you know this?
1
Not at all
2
3
4
5
Perfectly
39
Q
  1. Which statement is TRUE when comparing a functional restoration program to active individual therapy for chronic low back pain?
    (a) Flexibility is increased to a greater extent with active individual therapy program.
    (b) Pain intensity is reduced to a greater extent with active individual therapy.
    (c) Functional restoration programs have a greater effect on flexibility and pain than do active individual therapy programs.
    (d) Functional restoration programs produce greater improvements in endurance than do active individual therapy programs.
A

Answer: D

Commentary: Functional restoration programs produce a greater improvement in endurance, but no differences are noted between functional restoration programs and active individual therapy programs.

How well did you know this?
1
Not at all
2
3
4
5
Perfectly
40
Q
  1. 20-year-old college basketball player was seen in the training room after practice. He reports “twisting” his ankle while attempting to rebound a missed shot. On further questioning, he describes an inversion-type injury. He has swelling along the lateral aspect of the ankle. He is tender to palpation over the anterior talofibular ligament and calcaneofibular ligament as well as the 5th metatarsal base. He has no pain over the lateral or medial malleolus or proximally over the fibular head. You obtain plain radiographs, which show a nondisplaced avulsion fracture of the 5th metatarsal base. What is the next step in treating this individual?
    (a) Provide clearance for return to playing basketball without immobilization.
    (b) Obtain a surgical consult for possible screw or pin fixation.
    (c) Recommend immobilization with a postoperative shoe for 1-2 weeks.
    (d) Prescribe non-weight bearing with crutches for 6-8 weeks or until radiographically verified healing occurs.
A

Answer: C

Commentary: Nondisplaced or minimally displaced avulsion fractures of the 5th metatarsal base can occur with inversion ankle sprains. These generally are treated nonsurgically with a short course of immobilization (1-2 weeks) with a postoperative shoe or a short walking boot. Displaced fractures may require screw or pin fixation. It is important to differentiate an avulsion fracture of the base from a fracture of the metaphyseal-diaphyseal junction (Jones fracture), since treatment is different.

How well did you know this?
1
Not at all
2
3
4
5
Perfectly
41
Q
  1. You have evaluated a 50 year old man for lower extremity muscle pain and discomfort. The pain increases with jogging. You have reviewed his medications, which include simvastatin (Zocor). Baseline laboratory studies were normal 6 months ago. The creatine kinase level is mildly elevated at 185 units/L. The next most appropriate step is to
    (a) check thyroid stimulating hormone levels.
    (b) order electrodiagnostic study.
    (c) switch to a different class of lipid lowering medications.
    (d) continue the medication with close monitoring of the creatine kinase levels.
A

(d)

If a patient on a statin presents with muscle complaints, with or without creatine kinase (CK) elevations, other causes, including strenuous exercise or hypothyroidism, must be considered. If a patient initially has normal or only moderately elevated CK levels, the statin may be continued with close monitoring of symptoms and CK levels; however, if symptoms become intolerable or if the CK level is 10 times the upper limits of normal (ULN) or greater, the statin must be discontinued. If myositis is present or strongly suspected, the statin should be discontinued immediately. Early diagnosis and treatment of symptomatic CK elevations, including cessation of drug therapies potentially related to myopathy, can prevent progression to rhabdomyolysis. Symptoms and CK levels should resolve completely before reinitiating therapy, at a lower dose if possible. Asymptomatic elevation of CK at 10 times the ULN or greater should also prompt discontinuation of the statin. Consideration should also be given to discontinuation of statins before events that may exacerbate muscle injury, such as surgical procedures or extreme physical exertion.Needle electromyography abnormalities are uncommon in statin-induced myopathy. An EMG does not exclude statin-induced myopathy, because it primarily affects type 2 muscle fibers. Electromyography is not routinely performed or recommended unless the clinical presentation does not improve with statin discontinuation or if concern exists about other diagnoses.

How well did you know this?
1
Not at all
2
3
4
5
Perfectly
42
Q
  1. In assisting patients returning to their previous level of work, work hardening programs can achieve return-to-work rates of
    (a) less than 25%.
    (b) 25%–49%.
    (c) 50%–75%.
    (d) more than 75%.
A

(d)

Return-to-work rates of 77% can be achieved with work hardening programs. Poor outcome was associated with an increased number of treatments before the program, an increased length of time off from work; the patient’s having lower satisfaction with the program, and a lawyer being involved in the case.

How well did you know this?
1
Not at all
2
3
4
5
Perfectly
43
Q

. If a man injures his low back while on the job and is off work for 6 months, then the chance that he will return to work is

(a) 25%.
(b) 35%.
(c) 50%.
(d) 75%.

A

(c)

There is about a 50% chance of return to work when a worker who injures his low back on the job is off work for 6 months. The rate drops to 25% when the worker is off for 1 year, and is minimal is he is off for 2 years.

How well did you know this?
1
Not at all
2
3
4
5
Perfectly
44
Q
  1. What percentage of patients with whiplash-associated disorders develop chronic symptoms?
    (a) less than 25%
    (b) 25%–49%
    (c) 50%–75%
    (d) more than 75%
A

(b)

Up to 33% of individuals with symptoms from whiplash-associated disorders have chronic symptoms. Symptoms associated with whiplash-associated disorders include neck pain, arm pain, paresthesias, temporomandibular joint dysfunction, headache, dizziness, visual disturbances, and difficulty with memory and concentration.

Ref: Panagos A, Sable AW, Zuhosky JP, Irwin RW, Sullivan WJ, and Foye PM. Industrial medicine and acute musculoskeletal rehabilitation. 1. Diagnostic testing in industrial and acute musculoskeletal injuries. Arch Phys Med Rehabil 2007;88(3 Suppl):S5.

How well did you know this?
1
Not at all
2
3
4
5
Perfectly
45
Q
  1. Which factor is most likely to be associated with the development of a work-related, repetitive-strain injury?
    (a) Normal body weight
    (b) Warm work environment
    (c) Younger age
    (d) Rheumatoid arthritis
A

(d)

Risk factors associated with a repetitive strain injury include obesity, cold temperature, older age, diabetes, smoking, pregnancy, rheumatoid arthritis, and psychologic stress.

Ref: Panagos A, Sable AW, Zuhosky JP, Irwin RW, Sullivan WJ, Foye PM. Industrial medicine and acute musculoskeletal rehabilitation. 1. Diagnostic testing in industrial and acute musculoskeletal injuries. Arch Phys Med Rehabil 2007;88(3 Suppl):S5.

How well did you know this?
1
Not at all
2
3
4
5
Perfectly
46
Q
  1. A 17 year old woman was involved in a motor vehicle crash 4 months ago. She suffered a shoulder dislocation. Electromyographic studies have confirmed a brachial plexus injury to her posterior cord and indicate nerve continuity (Sunderland 2 injury). Although she has completed 4 weeks of occupational therapy, she has had no improvement in her strength from baseline. Your next step would be to
    (a) reassure the patient and continue to monitor for improvement.
    (b) continue occupational therapy for 4 additional weeks.
    (c) initiate neuromuscular electrical stimulation to the affected muscles.
    (d) refer the patient to neurosurgery for exploratory surgery.
A

(d)

With closed nerve injury as described, early active and passive range of motion (ROM) therapy of the affected joints is begun. The value of electrical stimulation is uncertain. The purpose of surgical repair is to improve peripheral nerve recovery and eventual function. Therefore, surgery is done when the patient has an incomplete loss of function but shows no improvement over several weeks, or no return of function at 2 months for peripheral nerve and 4 months for a brachial plexus injury. Findings at the time of surgery help establish a prognosis. However, the chances of successful surgical repair begin to decline by 6 months after the injury. By 18–24 months, the denervated muscles usually are replaced by fatty connective tissue, making functional recovery impossible.

How well did you know this?
1
Not at all
2
3
4
5
Perfectly
47
Q

Which nerve does NOT innervate the outer annulus of the lumbar intervertebral disc?

(a) sinuvertebral nerve
(b) lumbar medial branches of dorsal rami
(c) grey rami communicantes
(d) lumbar ventral rami

A

(b)

The lumbar medial branches of the dorsal rami supply the facet joints as well as the deep paraspinals, such as the rotators and multifidi. The sinuvertebral nerve, also termed the recurrent meningeal nerve is the primary source of nerve supply to the lumbar intervertebral disc. It is derived from portions of the ventral rami and grey rami communicantes (sympathetic input). Accordingly, the referral pattern seen with intrinsic disc pain is vague and diffuse.

How well did you know this?
1
Not at all
2
3
4
5
Perfectly
48
Q
  1. A 22-year-old female gymnast with chronic low back pain is diagnosed with spondylolysis of the right L5 pars interarticularis. Spondylolisthesis is not identified on plain x-rays. What is the best test to determine fracture healing?
    (a) magnetic resonance imaging
    (b) single photon emission computed tomography
    (c) computed tomography scan
    (d) flexion and extension lateral x-rays
A

(c)

Computed tomography (CT) scans with thin cuts through the area of the pars interarticularis can identify the healing pattern of a pars stress fracture.

Ref: Standaert CJ, Herring SA. Expert opinion and controversies in sports and musculoskeletal medicine: the diagnosis and treatment of spondylolysis in adolescent athletes. Arch Phys Med Rehabil. 2007. Apr;88(4):537-40.

How well did you know this?
1
Not at all
2
3
4
5
Perfectly
49
Q
  1. Which factor increases the risk for long-term symptoms after a whiplash-type injury?
    (a) Male gender
    (b) Eastern European descent
    (c) Preexisting hyperlordosis of cervical spine
    (d) Presence of radiating pain into the limb
A

(d)

Risk factors for chronic whiplash-associated pain include presence of preexisting degenerative disc disease, preexisting loss of cervical lordosis, female gender, awkward head position at time of impact, presence of radiating pain into upper limbs, and prior history of headache. A famous Lithuanian study showed no incidence of long-term whiplash pain in a country that had no compensation system for whiplash.

Ref: Seroussi RE, Ferrari R. Curve/countercurve: Whiplash. SpineLine 2001:12-9.

How well did you know this?
1
Not at all
2
3
4
5
Perfectly
50
Q

You see the significant other of a close friend in your office for knee pain. As part of her past medical history you note that she has a congenital heart defect. She says she has not yet told your friend that she has this condition. You decide to tell your friend about her congenital heart defect even though the patient did not give you permission to do so. What penalty do you face for knowingly disclosing individually identifiable health information, which is in violation of HIPAA rules?

(a) $50,000 and up to 1year of imprisonment
(b) No penalty
(c) $250,000 and up to 10 years imprisonment
(d) $100,000 and up to 5 years imprisonment

A

(a)

A person who knowingly discloses individually identifiable health information in violation of HIPAA faces a fine of $50,000 and up to a 1-year imprisonment. The criminal penalties increase to $100,000 and up to 5 years imprisonment if the wrongful conduct involves false pretenses, and to $250,000 and up to 10 years imprisonment if the wrongful conduct involves the intent to sell, transfer, or use individually identifiable health information for commercial advantage, personal gain, or malicious harm.

How well did you know this?
1
Not at all
2
3
4
5
Perfectly
51
Q
  1. Which route of epidural steroid administration is most likely to deliver steroid to the junction of the posterior disc and anterior dura?
    (a) Transforaminal
    (b) Caudal with catheter
    (c) Interlaminar
    (d) Caudal
A

(a)

The subpedicular transforaminal route of epidural steroid delivery places the needle at the anterior portion of the intervertebral foramen. The retroneural route of delivery purposefully terminates needle placement at the posterior edge of the intervertebral foramen to avoid injuring radicular vasculature. The caudal and interlaminar approaches are of limited utility in delivering steroid anteriorly due to raphe within the epidural space.

Ref: Irwin RW, Zuhosky JP, Sullivan WJ, et al. Interventional procedures for work-related lumbar spine conditions. Arch Phys Med Rehabil 2007;88(Suppl):S22-3.

How well did you know this?
1
Not at all
2
3
4
5
Perfectly
52
Q

Under the Health Insurance Portability and Accountability Act (HIPAA) Protected Health Information is data that

(a) a physician can withhold from a patient.
(b) a patient’s job supervisor can obtain from a physician.
(c) can be used to identify a patient.
(d) can be shared with family without permission.

A

(c)

Protected Health Information includes individually identifiable health information. This is information, including demographic data, that relates to any of the following particulars: the individual’s past, present or future physical or mental health or condition; the provision of health care to the individual; or the past, present, or future payment for the provision of health care to the individual. It can also be information that identifies the individual or for which there is a reasonable basis to believe it can be used to identify the individual. Individually identifiable health information includes many common identifiers (e.g., name, address, birth date, Social Security Number).

How well did you know this?
1
Not at all
2
3
4
5
Perfectly
53
Q
  1. Which statement describes the natural history of lumbar spinal stenosis (LSS)?
    (a) The majority of individuals with LSS will develop focal weakness.
    (b) Of patients treated nonsurgically, 25%–50% have satisfactory outcomes.
    (c) Ambulation worsens in the majority of individuals with LSS.
    (d) Early surgery improves long-term outcome.
A

(b)

The natural history of spinal stenosis is generally benign. While decompressive surgery achieves satisfactory results in the great majority of individuals, the difference in outcomes with their nonsurgical cohorts becomes narrower with time.

How well did you know this?
1
Not at all
2
3
4
5
Perfectly
54
Q
  1. In 2003, which diagnosis-related group (DRG) had the most admissions to inpatient rehabilitation facilities?
    (a) Stroke
    (b) Unilateral joint replacement in a lower extremity
    (c) Amputation for circulatory disorders except upper limb and toe
    (d) Hip or pelvis fracture
A

(b)

In fiscal year 2003, the number of admissions to an inpatient rehabilitation facility with the diagnosis related group unilateral joint replacement in a lower extremity was 124,754, stroke was 54,433, amputation for circulatory disorders except upper limb and toe was 7,200, and hip or pelvis fracture was 5,863.

Ref: US Government Accountability Office. Medicare: more specific diagnoses needed to classify inpatient rehabilitation facilities. April 2005. Report GAO-05-366. Available at: http://www.gao.gov/new.items/d05366.pdf. Accessed July 4, 2007.

How well did you know this?
1
Not at all
2
3
4
5
Perfectly
55
Q
  1. You are called onto a football field immediately after a defensive player involved in a spearheading tackle complains of neck pain and right greater than left arm tingling. What should be the next step?
    (a) Call for an ambulance and stabilize the neck.
    (b) Remove the athlete’s football helmet and palpate for any neck tenderness.
    (c) Return the athlete to the game if his strength exam is normal.
    (d) Walk the athlete to the locker room and perform a thorough neurologic examination.
A

A telltale sign of cervical cord involvement is bilateral symptoms. In this case, the athlete should be treated as having a potential spinal cord injury and should have his cervical spine immobilized. The football helmet should not be removed, since the cervical spine may fall into extension in the act of removing the helmet. If the airway needs to be accessed, then the face guards should be removed using special equipment. If the athlete suffered and recovered from a temporary “stinger,” involving 1 limb, he may return to play as long as his neurologic examination is normal.

How well did you know this?
1
Not at all
2
3
4
5
Perfectly
56
Q
  1. A 42-year-old car mechanic with a 3-week history of low back pain and lower limb pain after lifting equipment at work is referred to you for management. He has been taking ibuprofen 800mg 4 times daily without improvement. He is unable to flex through the lumbar spine or sit without pain. Your recommendations to his employer regarding work include
    (a) modified duty to allow no repetitive twisting or bending and no push/pull heavier than 20 lbs.
    (b) return to sedentary work 8 hours daily for 1 week, and no push/pull heavier than 10 lbs.
    (c) light duty to include no pushing/pulling, or lifting more than 25 lbs for 1 month.
    (d) remain off work until lumbar flexion, sitting, and lifting are no longer painful.
A

Returning the employee to modified duty that fits the impairment and avoids provocative activities is important from several aspects. One, behavioral management with the employee allows early goals to be set, so that the employee can work with restrictions. It also establishes that simply being off work until pain free is not always a logical goal. Second, the employer can fully understand the employee’s capabilities during recovery. This management approach hones in on the employer to comply with the restrictions. Third, starting with reasonable restrictions allows the physician to guide the employee back to the work place by making adjustments as the worker’s rehabilitation progresses.

How well did you know this?
1
Not at all
2
3
4
5
Perfectly
57
Q
  1. A 55-year-old paramedic is under your care for a work-related shoulder injury. She has completed physical therapy, no longer requires pain medications, and wants to return to work. She does not have full shoulder abduction and has some pain with overhead activities. Ideally, you recommend
    (a) return to work without restrictions.
    (b) work conditioning for 4 weeks.
    (c) a week of work hardening.
    (d) functional capacity evaluation.
A

(d) The paramedic has a high demand job. A functional capacity evaluation would best determine the employee’s ability to return to her job. If deficits are noted, work hardening over a period of weeks will best ensure return to work. Work hardening for 1 week may not be sufficient. Work conditioning enhances aerobic fitness and conditioning but is not job specific. The paramedic is at high risk for recurrent injury. Returning the employee to work without testing the her ability to perform her job duties may precipitate premature return and reinjury.

How well did you know this?
1
Not at all
2
3
4
5
Perfectly
58
Q
  1. An 18-year-old, right-handed hockey player presents to you after experiencing 3 right shoulder anterior dislocations in the prior season after falls on ice. Magnetic resonance imaging shows supraspinatus tendonitis but no other lesions or tears. After 6 sessions of physical therapy, he is pain free. He has been invited to play professionally in 6 months. What is your next recommendation?
    (a) Tell him that he will likely dislocate again and that he should relocate the shoulder by forcefully pushing the anterior shoulder against a wall.
    (b) Refer him to a surgeon to consider shoulder stabilization surgery.
    (c) Tell him he should not return to any sports because of his increased chance of dislocating again.
    (d) Stress the importance of compliance with his home exercise program.
A

B
Recurrent dislocations should be treated with surgery at some point if the athlete would like to return to contact sports. Various anterior shoulder dislocation techniques that can be applied to reduce the shoulder, most by external rotation of the shoulder or by using gravity.

How well did you know this?
1
Not at all
2
3
4
5
Perfectly
59
Q
  1. [ITEM WAS NOT SCORED ON 2007 SAE-R]

You have evaluated a 50-year-old man for lower extremity muscle pain and discomfort. The pain increases with jogging. You have reviewed his medications, which include simvastatin (Zocor). Baseline laboratory studies were normal 6 months ago. The creatine kinase level is mildly elevated at 185 units/L. The next most appropriate step is to

(a) discontinue the medication and check creatinine and thyroid stimulating hormone levels.
(b) order electrodiagnostic study.
(c) switch to a different class of lipid lowering medications.
(d) continue the medication with close monitoring of the creatine kinase levels.

A

(d) If a patient on a statin presents with muscle complaints, with or without creatine kinase (CK) elevations, other causes, including strenuous exercise or hypothyroidism, must be considered. If a patient initially has normal or only moderately elevated CK levels, the statin may be continued with close monitoring of symptoms and CK levels; however, if symptoms become intolerable or if the CK level is 10 times the upper limits of normal (ULN) or greater, the statin must be discontinued. If myositis is present or strongly suspected, the statin should be discontinued immediately. Early diagnosis and treatment of symptomatic CK elevations, including cessation of drug therapies potentially related to myopathy, can prevent progression to rhabdomyolysis. Symptoms and CK levels should resolve completely before reinitiating therapy, at a lower dose if possible. Asymptomatic elevation of CK at 10 times the ULN or greater should also prompt discontinuation of the statin. Consideration should also be given to discontinuation of statins before events that may exacerbate muscle injury, such as surgical procedures or extreme physical exertion.

Needle electromyography (EMG) abnormalities are uncommon in statin-induced myopathy. An EMG does not exclude statin-induced myopathy, because it primarily affects type 2 muscle fibers. Electromyography is not routinely performed or recommended unless the clinical presentation does not improve with statin discontinuation or if concern exists about other diagnoses.

How well did you know this?
1
Not at all
2
3
4
5
Perfectly
60
Q
  1. Under the prospective payment system for inpatient rehabilitation facilities, which item is used in assigning a patient to a case-mix group?
    (a) Mini Mental Status Examination
    (b) Disability Rating Scale
    (c) Previous hospitalization
    (d) FIM instrument motor score
A

D
The prospective payment system for inpatient rehabilitation facilities requires that all patients admitted for inpatient rehabilitation be assigned to an impairment group code category. Payment to the rehabilitation facility is further determined by the patient’s subclassification into a case-mix group. The FIM instrument motor score is used to help determine the case-mix group designation under the prospective payment system for inpatient rehabilitation facilities. None of the other options listed are used in this process.

How well did you know this?
1
Not at all
2
3
4
5
Perfectly
61
Q
  1. The interdisciplinary approach to patient care emphasizes
    (a) common patient and team goals.
    (b) discipline-specific goals.
    (c) concentration on specific clinical problems.
    (d) treatment by multiple team members.
A

A
The interdisciplinary approach to patient care emphasizes common patient and team goals rather than discipline-specific goals. The patient and family members should be included in the goal setting process. All team members must work in a collaborative way to facilitate achievement of goals. Team members must have an appreciation for all the issues that affect the patient rather than focusing on an isolated problem. Team communication is essential at all points in the rehabilitation process, not just when problems occur.

How well did you know this?
1
Not at all
2
3
4
5
Perfectly
62
Q

A case manager comes to your office accompanying the injured worker you are managing. The front desk person asks if you will see the case manager with the patient. You respond that

(a) case managers inhibit patient care and you don’t wish to speak with them.
(b) as requested by the patient you will see the case manager following the interview and examination.
(c) you will speak with the case manager after the patient signs a release of information.
(d) the case manager should always be present at the time of the patient’s interview and examination despite the patient’s request to avoid the case manager.

A

B
Case managers are shown to be beneficial liaisons between the physician and workers compensation carrier and their presence facilitates patient care. To be treated as a workers compensation case, the patient must give the carrier full access to his/her medical record. The employee treated under workers compensation cannot restrict the access of the case manager to the physician; however, discussions with the case manger should be done in the environment that the patient requests.

How well did you know this?
1
Not at all
2
3
4
5
Perfectly
63
Q

The purpose of the Health Insurance Portability and Accountability Act (HIPAA) is to

(a) ensure that a patient’s medical record is available to health care providers as directed by the patient.
(b) allow qualified physicians access to the patient’s medical record.
(c) allow a lawyer access to a medical record only if litigation is pending.
(d) prohibit the release of confidential health information to insurance carriers.

A

AThe purpose of the Health Insurance Portability and Accountability Act (HIPAA) is to ensure that a patient’s medical record remains private, but is available to health care providers as directed by the patient. A non-treating physician, lawyer, or insurance company may have access to the record with written authorization by the patient or guardian. There are no stipulations about a physician’s qualifications with regards to medical information access.

(a) Office for Civil Rights-HIPAA Privacy. Incidental uses and disclosures. Revised April 2003. [cited 2006 July 13] Available from URL: http://www.hhs.gov/ocr/hipaa/guidelines/incidentalud.pdf

How well did you know this?
1
Not at all
2
3
4
5
Perfectly
64
Q

A 55-year-old long-distance truck driver is recovering from a work related low back injury that occurred during lifting. The worker has completed 2 weeks of physical therapy and continues to have low back pain, lower extremity pain, and paresthesias. The employer calls you and is upset that you have restricted the worker from truck driving during the treatment phase, citing that “driving is sedentary work.” You recommend that the driver refrain from truck driving because
(a) a minimum of 4 weeks of physical therapy will be necessary to facilitate recovery.

(b) low back pain has been found to be more frequent in people exposed to whole body vibration.
(c) workers with low back pain should not sit while symptoms of radiculopathy are present.
(d) the employer is unlikely to follow the restrictions you recommend

A

B
Whole body-vibration is associated with increased frequency of low back pain. Some studies have found a correlation between increased frequency of disc protrusion and occupational driving. The exposure to vibration will likely facilitate continued symptoms in this worker, and relative rest is indicated during the initial stages of recovery. There is no predetermined length of physical therapy that is associated with recovery. Workers with low back pain and leg pain must learn to sit without increasing symptoms. Complete avoidance will not necessarily improve recovery and is not practical. The driver can likely perform some duties with restrictions. The employer has the responsibility to provide a job that meets the restrictions set by the physician. If the employer is unable to provide a job with these restrictions then the employee must remain off work.

How well did you know this?
1
Not at all
2
3
4
5
Perfectly
65
Q

To allow pronation of the foot, which 2 joints must have their axis of rotation in parallel?

(a) Lisfranc and talonavicular
(b) Subtalar and calcanocuboid
(c) Talocrural and subtalar
(d) Talonavicular and calcaneocuboid

A

D
The transverse tarsal joint, namely the talonavicular and calcaneocuboid joints, must have their joint axes in parallel to allow for a flexible midfoot and pronation. If the axes intersect, the midfoot becomes rigid, which enables proper supination.

How well did you know this?
1
Not at all
2
3
4
5
Perfectly
66
Q

A 17-year-old female was involved in a motor vehicle crash 4 months ago. She sustained a shoulder dislocation. Electromyographic studies have confirmed a brachial plexus injury to her posterior cord and indicate nerve continuity. Although she has completed 4 weeks of occupational therapy, she has had no improvement in her strength from baseline. Your next step would be to

(a) reassure the patient and continue to monitor for improvement.
(b) continue occupational therapy for 4 additional weeks.
(c) initiate neuromuscular electrical stimulation to the affected muscles.
(d) refer her to neurosurgery for exploratory surgery.

A

D
With closed nerve injury as described, early active and passive range of motion exercise of affected joints is begun. The value of electrical stimulation is uncertain. Surgery is done when there is an incomplete loss of function but no improvement over several weeks or no return of function at 2 months for peripheral nerve and 4 months for a brachial plexus injury. The purpose of surgical repair is to improve peripheral nerve recovery and eventual function. Findings at the time of surgery help establish a prognosis. However, the chances of successful surgical repair begin to decline by 6 months after the injury. By 18 to 24 months, the denervated muscles usually are replaced by fatty connective tissue, making functional recovery impossible.

How well did you know this?
1
Not at all
2
3
4
5
Perfectly
67
Q

Which statement describes the chronic-pain concept of “central sensitization”?

(a) The evoked response of A-delta fibers to subsequent input is amplified.
(b) The influx of sodium is fundamental to electrical signaling and subsequent generation of action potentials and excitatory postsynaptic potentials.
(c) A complex set of activation-dependent post-translational changes occurs at the dorsal horn, brainstem, and higher cerebral sites.
(d) The so-called “inflammatory soup,” rich in algesic substances, causes a lowering of threshold for activation and subsequent evoked pain.

A

C
Central sensitization is a complex set of activation dependent post-translational changes occurring at the dorsal horn, brainstem, and higher cerebral sites that sensitizes the central nervous system to further perception of pain. Wind-up is an amplified evoked response to repeated afferent inputs at the level of the dorsal horn.

How well did you know this?
1
Not at all
2
3
4
5
Perfectly
68
Q

When using local steroid injections in patients with tendinopathies

(a) injection into the tendon substance is optimal.
(b) minimum interval between injections is 2 weeks.
(c) select the finest needle that will reach the area.
(d) early postinjection local anesthesia is a complication.

A

C
It is advisable to select the finest needle that will reach the area. The injection should be peritendinous with avoidance of the tendon to prevent rupture. The minimum interval between injections should be at least 6 weeks. Early postinjection local anesthesia is not a complication of steroids, but it will occur if local anesthetic is mixed with the steroid.

How well did you know this?
1
Not at all
2
3
4
5
Perfectly
69
Q

The activity established as most predictive of developing a low back disorder is

(a) carrying an object at an increased horizontal distance from the body.
(b) lifting an object repeatedly at 20% less than the individual’s maximum lift capacity.
(c) repetitive sit-to-stand transitions with a weighted back pack.
(d) bending at knees rather than at the waist to lift an object.

A

A
The work by Marras and colleagues showed that increasing the horizontal distance from the trunk of an object being carried increased the risk of developing a low back disorder. This increase in distance increased the forces consistently on the anterior column of the spine. Although the other options can all place the worker at risk for a low back injury, only the increased carrying distance from the trunk has been shown to be the most predictive of a low back injury.

How well did you know this?
1
Not at all
2
3
4
5
Perfectly
70
Q

Which term describes a maladaptive pattern of drug use marked by increasing doses to achieve a similar pain relieving effect and a withdrawal syndrome?

(a) Dependence
(b) Addiction
(c) Craving
(d) Tolerance

A

A
Dependence is a maladaptive pattern of drug use marked by tolerance and a drug-class-specific withdrawal syndrome that can be produced by abrupt cessation, rapid dose reduction, decreasing blood levels of drug, or administration of an antagonist. Tolerance is a state of adaptation in which exposure to a drug induces changes that result in diminution of 1 or more of the drug’s effects over time. Addiction is a chronic biopsychosocial disease characterized by impaired control over drug use, compulsive use, continued use despite harm, and craving.

How well did you know this?
1
Not at all
2
3
4
5
Perfectly
71
Q

If the L3 and L4 medial branches of the dorsal rami are ablated, the patient will experience blocked afferents from the

(a) L5-S1 facet joint.
(b) L4-5 facet joint.
(c) L3-4 facet joint.
(d) L2-3 facet joint.

A

B
The medial branches of the dorsal rami supply innervation to the facet joints and the deep paraspinals, namely the segmental multifidi and rotators. The sacral multifidi are innervated by the sacral (rather than the lumbar) dorsal rami. Each lumbar medial branch innervates the facet joint at and below its derivation. The L4-5 facet joint is innervated by the L3 and L4 medial branches, derived from the L3 and L4 nerve roots.

How well did you know this?
1
Not at all
2
3
4
5
Perfectly
72
Q

In order to appropriately follow Medicare regulations for teaching physicians, when caring for a patient with a resident physician, the attending physician must

(a) review the chart and personally document his/her level of involvement in patient care, separate from documentation performed by the resident.
(b) examine the patient with the resident and co-sign the resident note.
(c) examine the patient and review the resident’s medical record documentation.
(d) examine the patient, review the resident’s documentation, and personally document involvement in the history, exam, and medical decision-making.

A

D
) In order to appropriately follow Medicare regulations for teaching physicians, when caring for a patient with a resident physician, the attending physician must see the patient, review the medical record documentation of the resident, and personally document involvement in key aspects of the history, exam, and medical decision-making. Documentation from the resident alone does not confirm the level of attending physician involvement. The attending physician documentation combined with the resident documentation can be used to determine the level of care provided and the appropriate level of billing.

How well did you know this?
1
Not at all
2
3
4
5
Perfectly
73
Q

Which spinal level has the greatest depth of posterior epidural space?

(a) C3-4
(b) C4-5
(c) C5-6
(d) C6-7

A

C
he C6-7 and C7-T1 epidural levels have the greatest amount of space. Interlaminar epidural injections should be performed with caution in the spaces that have a smaller diameter, such as those at stenotic levels or high cervical levels. Practitioners should also be aware that the ligamentum flavum may have defects in a high percentage of individuals.

How well did you know this?
1
Not at all
2
3
4
5
Perfectly
74
Q

Which condition is a cumulative trauma disorder that has been associated with intensive computer use?

(a) Herniated thoracic disc
(b) Shoulder adhesive capsulitis
(c) Post-traumatic stress syndrome
(d) Cervical myofascial pain

A

D

The United States Department of Labor has determined that computer work is associated with a significant number of musculoskeletal disorders, many of which are considered cumulative trauma disorders. Examples include cervical and thoracic myofascial pain, rotator cuff tendonitis, medial and lateral epicondylitis, de Quervain tenosynovitis, and carpal tunnel syndrome.

How well did you know this?
1
Not at all
2
3
4
5
Perfectly
75
Q

. Professionalism is the basis of medicine’s contract with society. Which item is a fundamental principle of medical professionalism?

(a) Social justice
(b) Physician paternalism
(c) Patient disclosure
(d) Free enterprise

A

A
According to the Charter on Medical Professionalism, there are 3 fundamental principles of medical professionalism. They are (1) the primacy of patient welfare, (2) patient autonomy, and (3) social justice.

How well did you know this?
1
Not at all
2
3
4
5
Perfectly
76
Q

Practice-based learning and improvement is considered by the Accreditation Council of Graduate Medical Education (ACGME) to be an aspect of medical practice in which all physicians need to achieve and maintain competency. Which characteristic is NOT a key aspect of practice-based learning and improvement?

(a) The ability to locate, appraise, and assimilate evidence from scientific studies related to their clinical practice
(b) The ability to access and use information technology to support their own education
(c) The ability to apply knowledge of study designs and statistical methods to the appraisal of medical literature
(d) The ability to advocate for quality patient care and assist patients in dealing with system complexities

A

D
All of the options listed are key aspects of practice-based learning and improvement, with the exception of the ability to advocate for quality patient care and assist patients in dealing with system complexities. This statement is a key aspect of systems-based practice as defined by the Accreditation Council of Graduate Medical Education.

How well did you know this?
1
Not at all
2
3
4
5
Perfectly
77
Q

Which treatment has NOT been shown to improve epicondylitis?

(a) Low intensity laser irradiation
(b) Wrist extension strengthening exercises
(c) Acupuncture
(d) Extracorporeal shock-wave therapy

A

A
Wrist strengthening, acupuncture, and shock wave therapy all help in the treatment of epicondylitis. However, low intensity laser treatment is not proven beneficial.

How well did you know this?
1
Not at all
2
3
4
5
Perfectly
78
Q

The Commission on Accreditation of Rehabilitation Facilities (CARF)

(a) requires mandatory surveys of all inpatient rehabilitation facilities.
(b) provides accreditation status that confers a preferred status with payors.
(c) provides accreditation status that signifies the rehabilitation facility holds itself to the highest standards in the field.
(d) provides accreditation for comprehensive inpatient rehabilitation programs, but not specialty programs in areas such as spinal cord injury.

A

C
The Commission on Accreditation of Rehabilitation Facilities (CARF) provides accreditation status that signifies the rehabilitation facility holds itself to the highest standards in the field. CARF accreditation is voluntary and not all inpatient rehabilitation facilities participate. Accreditation by CARF does not confer any preferred status with payors, and CARF provides accreditation in general comprehensive inpatient rehabilitation as well as specialty programs such as spinal cord injury and traumatic brain injury.

79
Q

What is the most common level of occult spine fracture after trauma that is missed by plain

radiographs?

(a) C7/T1
(b) T5/T6
(c) T12/L1
(d) L4/L5

A

(a) Occult cervical fractures are most often seen at the C1 and C7 levels. By adding computed

tomography (CT) scanning to the evaluation of trauma patients, a significant number of occult

cervical fractures can be diagnosed. Of spinal fractures, 5% to 30% are multiple and may appear at noncontiguous levels. Thus, radiographic evaluation of the entire spinal axis is necessary wheneverinjury at 1 region of the spine is detected

80
Q
  1. An injured worker with low back pain will return to full work duties faster if treatment includes
    (a) back school and lumbar corset.
    (b) aerobic conditioning and weight lifting.
    (c) work conditioning and “off-duty” or no-work status.
    (d) functional rehabilitation and ergonomic intervention.
A

(d) Loisel, in1997, performed a randomized controlled trial that showed that the injured worker with

low back pain returned to regular work activities 2.41 times faster if the worker received

therapeutic functional rehabilitation and ergonomic intervention when compared to usual care.

81
Q
  1. Which statement is TRUE regarding the way the Centers for Medicare and Medicaid Services currently reimburses inpatient rehabilitation facilities (IRFs) based on a prospective payment system (PPS)?
    (a) Reimbursement is determined according to the patient’s severity of disability and his/her

required use of resources.

(b) Assignment of patients to a specific rehabilitation impairment category (RIC) is based

primarily on their medical co-morbidities.

(c) Early transfer of patients from an IRF to a skilled nursing facility does not affect

reimbursement to the IRF.

(d) Assignment of patients to specific case-mix groups (CMGs) is determined by the rehabilitation diagnosis and the patient’s premorbid functional status.

A

(a) The Center for Medicare and Medicaid Services currently reimburses inpatient rehabilitation

facilities (IRF) based on a prospective payment system (PPS) according to the patient’s severity of disability and his/her required use of resources. The rehabilitation impairment category is based on the primary rehabilitation diagnosis, and the case-mix group is determined in part by the patient’s co-morbid medical conditions

82
Q
  1. A 30-year-old runner training for a marathon presents to your clinic with progressive pain in his right medial tibia over the past 2 weeks. Which history and examination feature is more consistent with a stress fracture rather than medial tibial stress syndrome?
    (a) Diffuse rather than focal tenderness
    (b) Pain with percussion around the site of pain
    (c) Pain that diminishes as the run goes on
    (d) No reproduction of pain with single leg hop test
A

(b) This is a common clinical conundrum. Medial tibial stress syndrome (frequently and improperly referred to as shin splints) represents a range of pathology from posterior tibialis tendonitis to periostitis. Fredericson et al determined that stress fractures, which will require more aggressive treatment, have more focal pain and pain with percussion testing. Pain from a stress fracture worsens as run goes on and often hurts after a run. A magnetic resonance image or bone scan can make a definitive diagnosis.

83
Q

Spinal instrumentation in neuromuscular scoliosis is indicated when

(a) onset of scoliosis is before skeletal maturity.
(b) primary curve exceeds 25° and forced vital capacity (FVC) is greater than 35% of normal.
(c) ambulation abilities are lost due to severe back pain.
(d) primary curve exceeds 50° and FVC is less than 25% of normal.

A

(b) The indication for spinal instrumentation in neuromuscular scoliosis is that the primary curve

exceeds 25° and forced vital capacity has not dropped below 35% of normal.

84
Q

You are managing the care of a 63-year-old pipe fitter with shoulder pain in his dominant arm. He

has no history of trauma. An incomplete supraspinatus tear is found on imaging studies. He has

completed 8 sessions of physical therapy with improvement. He has pain at 140° of shoulder

abduction. Strength, range of motion, and neurological exam are normal. The case manager asks for

your next recommendation towards determining his work status. You recommend

(a) functional capacity evaluation exam.
(b) work conditioning.
(c) aquatic physical therapy.
(d) surgical referral.

A

(a) A functional capacity evaluation (FCE) is the next best choice for this worker. An FCE will

determine functional deficits the worker has by assessing work simulated activities. Work

conditioning may be helpful because it includes aerobic conditioning but does not provide

information and training regarding specific work tasks. His physical findings are minimal so

additional physical therapy may not be recommended. These minimal physical exam findings in a 63-year-old man may indicate a nontraumatic incomplete cuff tear that may be a part of normal

aging. In this setting, conservative management remains appropriate. Incomplete rotator cuff tears can be found in the asymptomatic population, especially over the age of 50.

85
Q

Medical error reporting systems are designed to

(a) ensure that patients and families are notified when a medical error has occurred.
(b) assist patients and families in reporting activities that they perceive as an error.
(c) discipline staff who report that an error has occurred.
(d) encourage staff to report errors without fear of punishment.

A

(d) Medical error reporting systems are designed to encourage staff to report sentinel events, adverse events, and close calls without fear of punishment. If these issues are recognized, then further review and action can be initiated. Review may include a root cause analysis to determine the exact cause of the problem and strategies for prevention. When a medical error has occurred, staff are encouraged to recognize the issue and report the issue immediately.

86
Q

You are managing the care of an injured worker with a working diagnosis of C6 radiculopathy. Your assistant informs you that the case manager is in the room with the patient. You next ask the

(a) case manager to leave, because case managers represent only the interest of the company.
(b) worker if he prefers to have the case manager present throughout the exam.
(c) worker if he prefers you recommend to the case manager that he be placed in a new job.
(d) case manager to show evidence that the worker is malingering.

A

(b) Case managers work as medical liaisons between the company and medical care team to facilitate

communication of and approval for diagnostic tests and treatment. They have been shown to

improve timeliness of care. It is important to ask the patient’s preference regarding the case

manager’s presence and to ensure patient confidentiality. The physician is to determine if the

patient can return to work based on objective and subjective evidence. It is not the case manager’s

job to provide the treating physician with evidence regarding validity of the worker’s condition.

87
Q
  1. Which action is a sacroiliac joint provocation test?
    (a) Thomas test
    (b) Gillet test
    (c) Thigh thrust maneuver
    (d) Flexion, adduction, internal rotation (FADIR) maneuver
A

(c) While data on sacroiliac joint provocative tests have been equivocal in the literature, recent

evidence has shown thigh thrust and pelvic distraction maneuvers to be helpful. The Thomas test is

used to assess hip flexor contracture. The Gillet test is a sacroiliac joint motion test and not a

provocation test. The FADIR maneuver is used to assess piriformis tightness and provocation.

88
Q
  1. Which condition is a proven barrier for the injured worker to return to a modified work position?
    (a) The employee has no incentive to return to modified work.
    (b) Employee and employer cannot agree on the job assigned.
    (c) The employee’s education does not match work requirements.
    (d) The employer must pay a fee to the carrier to implement modified work
A

(c) Return-to-work barriers proven to date include the following: lack of knowledge regarding

modified work, negative attitudes of employees, difficulty changing the work tasks and

organization of the work, and a mismatch between the employee’s education level and the

requirements of the modified job.

89
Q
  1. When performing an intra-articular facet joint injection on a patient with no previous history of

surgery, you note extravasation of dye, which is flowing anteriorly into the epidural space. A defect

in what structure will cause this scenario?

(a) Posterior facet capsule
(b) Ligamentum flavum
(c) Interspinous ligament
(d) Dural sac

A

(b) The ligamentum flavum forms the anterior border of the facet joint. Rents in the facet capsule

allow contrast material to leak and therefore medication will also leak out of the intended area.

Intra-articular injections into the facet joint performed for diagnostic purposes may lose specificity

when this extravasation into the epidural space occurs, because other pain generating structures

may also be blocked from receiving the medication.

90
Q
  1. Which behavior would most likely be a warning sign of substance abuse in a resident colleague?
    (a) Deterioration in personal hygiene
    (b) Infrequent tardiness to scheduled lectures
    (c) Excessive concern regarding patient well-being
    (d) Frustration over evening admissions to the rehabilitation unit
A

(a) Deterioration in personal hygiene is the most likely warning sign of substance abuse in a resident
colleague. Other warning signs of impairment secondary to substance abuse include increased rates

of absenteeism, inability to meet deadlines, loss of concern about patient welfare, and wide

fluctuations in mood and performance.

91
Q
  1. A 35-year-old receptionist presents with right lateral epicondylitis and periscapular myofascial pain.

What change should be made to her desk?

(a) Adjust chair height to allow 120° of knee flexion
(b) Move computer disk drive to the floor to facilitate seated flexion.
(c) Move mouse to allow 40° to 90° of elbow flexion.
(d) Change height of computer screen to allow neck extension.

A

(c) The mouse should be placed at a distance that is easy to reach and allows comfortable flexion of the elbow approximately 40° to 90°. Chair height is not a preset determined height but is

individualized so that the hips are at a 90° angle to the knees. The computer disk drive should be

easily reached if used repetitively. Avoiding seated flexion would be a better recommendation.

The computer screen height should allow a neutral neck position and minimize static flexion or

extension.

92
Q
  1. Which action is NOT required of a certified physiatrist to maintain certification?
    (a) Obtain continuing medical education credits.
    (b) Maintain active medical license.
    (c) Complete a recertification examination every 10 years.
    (d) Publish at least 1 article in a scientific journal every 10 years.
A

(d) Once a physician is certified by the American Board of Physical Medicine and Rehabilitation, he or she must continue to fulfill certain requirements in order to maintain certification status. Publication of an article in a peer-reviewed journal every 10 years is not a requirement for maintenance of certification. All the other options listed are required.

93
Q
  1. Which anatomical configuration is associated with an anterior pelvic tilt when the patient is

standing?

(a) Short abdominal muscles
(b) Increased lumbar lordosis
(c) Elongated hip flexors
(d) Short and strong gluteal muscles

A

(b) An anterior pelvic tilt is caused by one of several anatomic factors. These include tight hip flexors,

weak lower abdominals, and weak gluteal muscles. Compensatory lumbar extension through

lumbar lordosis results in individuals with excessive anterior pelvic tilt.

94
Q
  1. An overhead athlete with shoulder pain is found to have shoulder posterior capsule tightness. A biomechanical consequence of this tightness is
    (a) posterior displacement of the humeral head relative to the glenoid when the shoulder is
    flexing.
    (b) glenohumeral internal rotation deficit with range of motion less than 15° from neutral.
    (c) impingement of the humeral head onto the superior glenoid labrum when the shoulder is

abducted less than 90°.

(d)posterior tilting of the scapula in the static position and with shoulder abduction.

A

(b) A tight posterior capsule is a common phenomenon in overhead athletes. Often rotator cuff

tendinopathy results from impingement and scapular dyskinesis. Glenohumeral internal rotation

deficit (GIRD) defined as internal rotation less than 15° or a greater than 25° side to side difference,

is due to tight posterior capsule, teres minor, and infraspinatus. Further, Matsen and Harriman

showed that a tight posterior capsule drives the humeral head anteriorly when flexing the shoulder,

thus predisposing to further impingement at the coracoacromial arch. Scapular dyskinesis, due to

weak scapular stabilizer muscle weakness, is typically in a position of anterior tilting rather than

posterior tilting.

95
Q
  1. The purpose of the Health Insurance Portability and Accountability Act (HIPAA) is to
    (a) ensure that a patient’s medical record is available only to health care providers as directed by

the patient.

(b) make sure that only qualified physicians have access to a patient’s medical record.
(c) allow a lawyer access to a medical record only if litigation is pending.
(d) allow only the treating physician to know the patient’s diagnosis.

A

(a) The purpose of the Health Insurance Portability and Accountability Act (HIPAA) is to ensure that a

patient’s medical record remains private. A non-treating physician, lawyer, insurance company,

etc. can have access so long as the patient directs it, with written authorization. There are no

stipulations about a physician’s qualifications with regards to medical information access.

Office for Civil Rights–HIPAA Privacy: incidental uses and disclosures. Revised April 2003. Available

96
Q
  1. Which exercise creates the least stress of the rotator cuff musculature?
    (a) Arms in the scapula plane in internal rotation with weights
    (b) Arms in the scapula plane in external rotation with weights
    (c) Prone horizontal shoulder abduction in external rotation
    (d) Quadruped push-up
A

(d) The quadruped push-up is a closed kinetic chain (CKC) exercise. CKC exercises are the least

demanding of the rotator cuff exercises. The axial load with CKC exercises effectively reduces the

weight of the arm (ie, load). CKC promote muscle co-activation needed for functional tasks.

97
Q
  1. A 35-year-old female tennis player and data entry clerk presents with a 6-week history of lateral

elbow pain. Which factor is promoting her injury?

(a) Her strong scapular stabilizer muscle strength
(b) Her leading with the shoulder when performing the backhand tennis stroke
(c) Her keyboard being placed at too high a level at work
(d) Her tennis racquet string tension being recently increased from a low to a moderate string

tension.

A

(c) This patient has lateral epicondylitis or so-called tennis elbow. Weak scapular stabilizer muscle

strength may cause decreased force production through the elbow, thus causing more force and load

through the elbow. Leading with the elbow during the backhand stroke can cause an increased load

on the wrist extensor muscles. Keyboards placed too high or too low may cause lateral

epicondylitis. Racquet string tension should be adjusted to mild to moderate tension (52 to 55

pounds if stringing machine is properly calibrated) in a normal (non-oversized) racquet. A racquet

strung to less than 30 pounds will require the tennis player to use more force. This may, in turn,

cause increased grip pressures and wrist extensor overuse.

98
Q
  1. A 20-year-old female long-distance runner presents with a l-week history of worsening lateral hip pain. Magnetic resonance imaging reveals bone edema at the lateral aspect of the femoral neck, without evidence of a fracture line or cortical break. What is the next step in management?
    (a) Relative rest for 4 weeks
    (b) Refer to orthopedic surgeon for surgical evaluation
    (c) Order a triple-phase bone scan to determine acuity of the inju
    (d) Physical therapy with closed kinetic chain exercise emphasis
A

(b) Femoral neck stress fractures are serious injuries. Stress fractures may occur without a cortical break. Stress fractures of the medial side of the femoral neck, the “compression” side, can be treated with a weight-bearing restriction for 6 weeks or longer including no running or high impact activities. Stress fractures on the lateral side of the femoral neck, the “tension” side, warrants strong surgical consideration because of the poor healing capability at that location.

99
Q
  1. The term maximum medical improvement (MMI) indicates that
    (a) no deterioration of the condition is expected to occur.
    (b) no further treatment for the condition is required.
    (c) no further treatment is reasonably expected to improve the condition.
    (d) physician and patient agree that the condition has stabilized.
A

(c) Maximum medical improvement (MMI) is a term used by a physician to indicate that he/she has

determined that no further significant recovery of a condition is anticipated to occur. Resolution of

the condition may or may not occur. Determining MMI does not indicate that no treatment is

indicated

100
Q
  1. Which statement about workers’ compensation cases is TRUE?
    (a) The most recent publication of the American Medical Association guidelines for disability

are used uniformly in every state.

(b) Parameters that guide entry to evaluation and treatment are unique to each state.
(c) States require subspecialty board certification for physicians who practice industrial medicine.
(d) Most states allow injured workers to choose the physician who manages their care.

A

(b) Guidelines for worker entry and treatment under workers’ compensation law are determined by

each state. The published American Medical Association guidelines are useful tool in determining

disability. Because each state determines which edition of the publication is used, its application is

not uniform nationwide. Not all states require subspecialty certification required of a physician

wishing to practice industrial medicine. In some states the employer chooses the physician to treat

the injured worker.

101
Q
  1. A data entry operator presents to your office complaining of a 2-week history of bilateral elbow

pain that increases in severity the longer she types. She denies sensory changes or weakness. She

has not had treatment. The only significant exam finding is pain on palpation of the proximal

extensor forearm muscles just distal to the lateral epicondyles. You recommend

(a) ice, physical therapy, evaluation of keyboard placement.
(b) heat, extensor strengthening exercises, raising of keyboard height.
(c) ultrasound, elbow brace to limit flexion, raising of monitor height.
(d) iontophoresis, wrist splint to limit extension, lowering of monitor height.

A

(a) The most likely diagnosis in this case is lateral epicondylitis. In acute overuse injuries ice should

be used first to reduce inflammation. Physical therapy may be used to facilitate a home exercise

program for flexibility and strengthening exercises and to ensure appropriate posture adaptations.

The computer set-up—including chair height and positioning of the monitor, keyboard, and

mouse—will help resolve current complaints and reduce risk of re-injury. This ergonomic

evaluation is multifactorial.

102
Q
  1. A factory owner consults you to make recommendations on how to reduce workers’ compensation
    claims. You recommend evaluation of workers’ ergonomics and equipment. Educating the

employer regarding psychosocial factors is important because workers

(a) who do not enjoy their job are more likely to report a back injury.
(b) will claim low back injuries for attention.
(c) employed in factories often have a psychiatric history.
(d) claiming injuries after ergonomic evaluation are malingerers.

A

(a) Bigos’s study of over 3,000 aircraft employees identified risk factors for low back injury that are

outside of the physical factors. Workers who hardly ever enjoyed their job were 2.5 times more

likely to report a back injury than those who “almost always” enjoyed their job. This study

suggested that identifying factors outside of the physical and ergonomic model was important in

determining risk of injury. No studies demonstrate that factory workers often have mental illness.

103
Q
  1. During a physical examination, which structure CANNOT be effectively assessed by the modified

Thomas test?

(a) Iliotibial band
(b) Hip flexors
(c) Knee extensors
(d) Hamstrings

A

(d) The modified Thomas test is performed with the patient supine; with the tested leg in maximal

passive hip extension and knee flexion and the contralateral knee to the chest. The test position

causes the pelvis to be placed in posterior pelvic tilt and reduces any lumbar lordosis. Tightness of

hip flexors is measured by the degrees of hip flexion from neutral. Tightness of the knee extensors

is measured by the degrees of knee flexion from neutral. Tightness of the iliotibial band is

measured by the degrees of hip and knee external rotation.

104
Q
  1. Studies have shown that case managers working with injured workers have
    (a) interfered with the physician’s recommendations.
    (b) made recommendations that favor the insurer.
    (c) reduced workers’ compensation costs and lost-time cases.
    (d) interfered in the relationship between physician and patient.
A

(c) Case managers reduce workers’ compensation costs by 23% and reduce the overall number of

lost-time cases. Overall case management can help improve the quality of care, reduce cost, and

decrease time loss in the worker’s compensation system. Studies show that case managers doe not

interfere with the physician-patient relationship or the physician’s recommendations, and do not

make proposals that favor the insurer.

105
Q
  1. At the time of the initial evaluation of an injured worker, the physiatrist should
    (a) define the anticipated time frame that pain medications will be used.
    (b) ensure that the worker understands narcotic medications will be used if necessary up until

time to return to work.

(c) recommend that only non-narcotic pain medications will be used during rehabilitation.
(d) recommend patient-directed use of pain medications during rehabilitation and return to work.

A

(a) Narcotics and non-narcotic medications often may be necessary to manage pain in the injured
worker. It is important to set expectations of how these medications will be used. Expected length

of time of narcotic usage is especially important. The goal should be to return the worker to his/her

job without medication. Simply cutting off medications may be inappropriate. As pain improves,

and strength and function improve, medications should be tapered.

106
Q
  1. Which statement regarding the multifidi muscles of the back is TRUE?
    (a) Atrophy of the multifidi occurs in patients with low back pain.
    (b) Contraction of the multifidi causes gross trunk extension.
    (c) The multifidi muscles cross 4 or more spinal levels.
    (d) The multifidi have a low composition of muscle spindles.
A

(a) The multifidi are local postural muscles of the lumbar spine. They are length transducers or

position sensors of a spinal segment, by way of their rich composition of muscle spindles. The

multifidi pass along 2 or 3 spinal levels. They are theorized to work as segmental stabilizers rather

than producing gross trunk motion, since they do not have a large moment arm. Researchers have

found atrophy of the multifidi in people with low-back pain.

107
Q
  1. A 49-year-old, right-handed man who works at an automobile plant painting cars presents with a 4-

week history of right shoulder pain. His pain increases with overhead activities and with donning

his coat. He is unable to sleep due to pain at night. With regards to work, you recommend physical

therapy, ibuprofen and

(a) modifying work activities to reduce overhead activities.
(b) staying off work for 4 weeks.
(c) no work restrictions or modifications.
(d) staying off work until pain free with strengthening activities.

A

(a) Modifying the activity that is causing the repetitive overload is necessary if treatment is to succeed.

Treatment success is measured by reduced pain and return to function, in this case to an occupation

that requires repetitive overhead activities. The healthcare provider must communicate with the

employer to best understand the specific musculoskeletal job requirements and modify these

activities during the recovery phase of rehabilitation. Taking the worker off work entirely engages

disability behavior and may not serve a specific recovery function. Not making a work

modification is also a mistake, because the overload to the upper quadrant will continue and that

will impair rehabilitation.

108
Q

the medical literature supports the use of lumbar epidural steroid injections for which condition?

(a) Sclerotomal pain from facet-mediated low back pain
(b) Referred pain from a quadratus lumborum trigger point
(c) Neurogenic claudication due to lumbar spinal stenosis
(d) Radiculopathy caused by foraminal disc herniation

A

(d) Epidural steroid injections (ESIs) are indicated for radicular rather than referred pain. While

anectodotal descriptions of ESIs used for neurogenic claudication may present them as helpful, the

efficacy of ESIs have yet to be established for this condition.

109
Q

Once an individual becomes board-certified in the specialty of physical medicine and rehabilitation,

he/she must continue to fulfill certain requirements in order to maintain certification status. Which

action is NOT a requirement for maintenance of certification?

(a) Continuing medical education credits
(b) Maintenance of active medical licensure
(c) Completion of a recertification examination every 10 years
(d) Publication of at least 1 article in a scientific journal every 10 years

A

(d) Once an individual becomes board certified in the specialty of physical medicine and rehabilitation,

he/she must continue to fulfill certain requirements in order to maintain their certification status.

Publication of 1 article in a peer-reviewed journal every 10 years is not a requirement for

maintenance of certification. All of the other options listed are required.

110
Q

. You are asked to make specific exercise recommendations regarding how to reduce low back

injuries in a factory’s workers. As part of your program, you recommend that workers

(a) be fitted for a lumbar corsets.
(b) with heavy lifting work requirements avoid participation in sports.
(c) with a history of low back pain be moved to more sedentary, seated jobs.
(d) be instructed to reduce the amount of early morning flexion.

A

(d) Patients instructed to limit early morning flexion have shown reduced pain intensity, compared to
controls. This finding is related to disc hydration that occurs during sleep. No studies have shown

that all workers wearing lumbar corsets have reduction in injuries. Athletic activities outside of the

work environment often help improve endurance, which is protective for low back pain. Sedentary

111
Q

. A 45-year-old auto mechanic with a history of low back pain and a herniated disc presents to you

with 2-day history of leg pain, numbness in the lower extremity, stumbling, and 2 episodes of

urinary incontinence. Your recommendation includes

(a) Ibuprofen, ice, relative rest, pelvic tilts, and repeat magnetic resonance imaging this week.
(b) Lumbar epidural steroid injection and physical therapy.
(c) Immediate magnetic resonance imaging and referral to a spine surgeon.
(d) Acetaminophen, ultrasound, light duty for 5 days, and physical therapy.

A

(c) In this patient with known herniated disc now with urinary incontinence the primary concern should

be to evaluate for cauda equina syndrome. The MRI is necessary to determine if an extruded disc is

causing the current symptoms. Surgery in the acute setting is essential to achieve neurologic recovery

112
Q

. The owner of a landscape business is interested in implementing a program to reduce low back injuries in his workers. Which therapeutic exercise program has been shown to reduce the risk of low back injuries?

(a) Lumbar extension exercises
(b) Lumbar flexion exercises
(c) Lumbar spine stabilization exercises
(d) No specific group of exercises reduces risk

A

(d) No specific lumbar spine strengthening program has been shown to prevent low back pain in the

work place. Spine strength is not a predictor of reduced risk for onset of low back pain.

113
Q

How much knee flexion is required to descend stairs step over step after a total knee replacement?

(a) 45°
(b) 70°
(c) 90°
(d) 110°

A

D

Descending stairs requires 110° knee flexion.

114
Q

A 26-year-old mail handler is sent to you for management of her severe sensorimotor carpal tunnel

syndrome confirmed by electrodiagnostic evaluation. She was given a splint for her presumed

carpal tunnel syndrome 3 months ago, which she has worn 24 hours a day since that time without

relief. She notes severe tingling in her fingers that is worse at night, and she also notes difficulty

with gripping the mail, because of subjective weakness. She is now having severe pain, which

radiates up her hand into her forearm. You consider that a corticosteroid injection might benefit

this patient. Which statement is most correct regarding this injection?

(a) The risk of intraneural injection is too high, and the patient should not be injected.
(b) So that intraneural injection can clearly be recognized, do not dilute the corticosteroid with anesthetics.
(c) Persisting or worsening pain and numbness or swelling normally last for more than 48

hours postinjection.

Local tenderness and superficial hematomas are rare after this injection.

A

(b) The risk of intraneural injection is real, but in experienced hands this injection is safe. Anesthetics

mixed with the corticosteroid can mask the pain associated with needle placement into the nerve

and should not be used. Numbness is anticipated with this injection without use of anesthetics, and

helps to confirm proper placement. Local tenderness and hematomas are common with this

injection and do not represent a complication. Persistent or worsening pain or swelling lasting more

than 48 hours are signs of nerve injection or neurotoxic injury

115
Q

The presence of nonorganic physical signs in low-back pain patients as described by Gordon

Waddell is intended to

(a) alert the examiner that psychological issues may be contributing to pain.
(b) identify patients and workers that are malingerers.
(c) determine which workers would benefit from low back and abdominal exercises.
(d) indicate which patients require examination by a psychiatrist

A

(a) Nonorganic physical signs in patients with low back pain was described by Gordon Waddell. His

original paper describes the physical signs that correlated with psychological data and were

distinguishable from standard clinical signs of physical pathology. Waddell’s signs should be used

to determine if nonorganic issues may be contributing to the patient’s complaints. Not all patients

with nonorganic physical signs are malingerers or require psychiatric evaluation. The signs are not

indicators for therapeutic modalities.

116
Q

You prescribe a work-hardening program for a 36-year-old assembly-line worker. Work-hardening

programs are most effective when

(a) focused on upper limb function.
(b) the patient’s job functions are simulated.
(c) combined with a physical therapy program.
(d) prescribed by a physiatrist

A

b
Work-hardening programs are most effective when essential job duties are simulated.

Bonfilglio RP, LaBan MM, Taylor RS. Industrial rehabilitation medicine managemen

117
Q

Which steroid compound has the longest half-life?

(a) Dexamethasone
(b) Prednisone
(c) Triamcinolone
(d) Hydrocortisone

A

a
The least water-soluble compound will have the longest half-life.

Braddom RL, editor. Physical medicine and rehabilitatio

118
Q

A 32-year-old female cashier presents to you with a 2-week history of low back pain. She denies specific trauma or activity with onset. She has no lower extremity pain, numbness, or tingling. She denies night time pain, recent illness, or previous history of low back pain. Which radiological diagnostic test would you recommend?

(a) Computed tomography scan
(b) Radiographic imaging
(c) No imaging
(d) Magnetic resonance imaging

A

C
(c) No imaging is necessary in this patient with no known medical risk factors. The low back pain can

be managed initially without the expense of CT or MRI. Recommendations for obtaining x-rays for

the patient with a first episode of low back pain for less than 7 weeks include: over age 65, history

of osteoporosis, history of urinary tract dysfunction, persistent sensory loss, progressive pain

despite treatment, night or rest pain, fever, chills, unexplained weight loss, history of trauma or

repetitive overuse, recurrent pain with no x-rays in 2 years, previous surgery, or fracture.

119
Q

In an otherwise healthy worker with complaints of low back pain and no neurologic deficit on physical exam, magnetic resonance imaging of the lumbar spine is indicated when low back pain

(a) persists for 8 weeks.
(b) first occurs, so that work restrictions can be made.
(c) persists after 2 weeks of bed rest.
(d) continues for 4 weeks after the worker is placed on light duty.

A

(a) A magnetic resonance imaging (MRI) provides the most accurate and complete information during the initial assessment for lumbar disc pathology, stenosis, infection, tumor, or trauma. Because many patients who present with low back pain and lumbar radicular pain may have spontaneous improvement of symptoms, it is recommended to wait for approximately 7 weeks without improvement and with proper care before performing an MRI.

120
Q

What scapular motions should a patient avoid after a shoulder replacement?

(a) Internal rotation and adduction
(b) Internal rotation and abduction
(c) External rotation and adduction
(d) External rotation and abduction

A

(d) External rotation and abduction increase risk of dislocation because of anterior instability.

121
Q

A 52-year-old house painter has completed physical therapy for shoulder pain due to rotator cuff tendonopathy. He is independent with his home exercise program and pain free with his usual activities. He has been off work for 10 weeks. You next recommend

(a) return to work without restrictions.
(b) a functional capacity evaluation.
(c) physical therapy for a total of 14 weeks.
(d) an aerobic conditioning program.

A

(b) A functional capacity evaluation will determine if the worker can return to his work duties on a safe and dependable basis. A painter’s work responsibilities require significant repetitive upper

extremity activities not necessarily challenged with therapeutic exercise. Although aerobic

conditioning is important, simulation of work duties should be done before this patient returns to

work.

122
Q

Which factor is associated with the highest risk for postoperative dislocation after a total hip

arthroplasty?

(a) Femoral anteversion
(b) Anterior approach for hip replacement
(c) Long femoral component
(d) Previous hip replacement

A

(d) Patients with history of previous hip replacement and posterior approach are at greater risk for postoperative dislocation. Posterior surgical approach has a higher risk of dislocation than anterior

approach.

123
Q

A 62-year-old ice cream truck driver fell getting out of his truck and suffered a medial meniscal

tear. After 4 weeks of physical therapy, he has 95° of knee flexion, and the knee effusion has
resolved. He is able to perform his activities of daily living with minimal pain. A functional

capacity evaluation determines that he is able to perform duties at a medium level category. His

job requires him to lift 75 pounds at each delivery site. You recommend

(a) return to work with a lifting restriction of 20 pounds.
(b) continue in physical therapy to increase knee flexion to 115°.
(c) undergo a work hardening program with re-evaluation in 3 weeks.
(d) return to work with a derotation knee brace.

A

(c) The functional capacity evaluation indicated he can lift up to 50 pounds safely but his job requires

that he lift 75 pounds so further intervention before return to work is indicated. A work hardening

program is an interdisciplinary program that will incorporate work-specific activities and

conditioning to transition the worker from acute care to a safe return to work. Although gaining

knee flexion may be beneficial, it is not the major focus for return to work.

124
Q

Which factor contributes most to poor worker motivation during rehabilitation?

(a) Loss of worker identity
(b) Boredom while away from work
(c) Good evaluations by a supervisor prior to injury
(d) Worker’s perception of severity of pain

A

(a) Loss of worker identity caused by depression and anxiety is associated with poor motivation during

the rehabilitation process. Other associated factors include deteriorating financial status, change in

family roles, loss of control regarding the future, and deteriorating employer relationships.

125
Q

A 25-year-old construction worker fell from a ladder this morning. He presents with knee pain, immediate swelling, and inability to bear weight on the injured leg. The best initial

recommendations would be

(a) hydrocodone, a knee brace, and return to work with light duty restrictions.
(b) nonsteroidal anti-inflammatory medication, x-ray, and return to work with a climbing

restriction

(c) ice, crutches, x-ray, and sedentary work restriction.
(d) tramadol, ace wrap, and off work for 1 week.

A

(c) This worker has sustained a significant injury and may likely have torn his anterior cruciate
ligament. The most appropriate recommendations in the acute setting include ice, compression,

elevation, and x-ray to rule out fracture. Work restrictions should allow the worker to remain with restricted weight bearing until improvement is noted or further testing is completed.

126
Q

What is a barrier for return to work for the injured worker?

(a) Receiving a portion of salary while recovering
(b) Having a good relationship with supervisor
(c) All lost salary being recoverable upon return to work
(d) Being married to a spouse who is employed

A

a) Financial barriers that are disincentives for return to work include that the worker receives a portion

of his/her salary while not working and that the worker has potential for monetary gain through

legal settlement.

127
Q

Which biomechanical factor is NOT implicated in persons at risk for recurrent stress fractures?

(a) Excessive supination of the foot
(b) Forefoot valgus
(c) Leg length discrepancy
(d) Hip joint hypermobility

A

(d) Reduced (not excessive) hip rotation, along with excessive supination or pronation of the foot,

forefoot varus or valgus, and leg length inequality have all been implicated in those at risk for

recurrent stress fractures.

128
Q

What is the most common reason for revision of a total hip arthroplasty?

(a) Recurrent dislocation
(b) Peri-prosthetic fracture
(c) Aseptic loosening from polyethylene wear
(d) Infection

A

(c) Aseptic loosening caused by polyethylene wear is is the most common reason for hip arthroplasty

revision surgery.

129
Q

A 39-year-old factory line worker admits to not wanting to return to work. Which job characteristic is associated with a delayed return to work?

(a) Evening shift
(b) Job monotony
(c) Autonomy
(d) Flexible hours

A

(b) Several job characteristics are associated with delayed return to work. These characteristics include:

low pay, low c

130
Q
  1. Which finding on your history and physical examination of an injured worker with low back pain would require immediate intervention?
    (a) Pain radiating into the posterior thigh
    (b) Pain that awakens the worker at night
    (c) Paresthesias in the great toe
    (d) Ankle weakness only with long-distance walking
A

(b) Pain that awakens the worker at night may indicate a serious etiology including tumor or infection.

Radicular pain, paresthesias, and weakness with fatigue may also present with serious as well as

common causes such as disc herniations and spinal stenosis. The latter may help direct the

diagnosis. However, pain worse at night should alert the treating physician to evaluate more serious etiology without delay.

131
Q

hy is it important to ask the injured worker if he/she is involved in litigation against the

workplace?

(a) It allows the physician to deny seeing the patient to avoid a deposition.
(b) It is not an appropriate question for the physician to ask.
(c) Workers with attorneys involved are less likely to return to work.
(d) The employer will be more cooperative in facilitating return to work.

A

(c) Several studies have found a strong relationship between attorney involvement and not returning to work. In workman’s compensation cases involving lawyers, 73% of injured workers did not return to work. In cases without lawyer involvement, 32% did not return to work. These factors are important for the physician to know initially to best understand the psychosocial barriers involved in treating the injured worker.

132
Q

Which situation is associated with good treatment outcome in the injured worker?

(a) Poor evaluation by the employer within the past 6 months
(b) The employee works in middle management
(c) The employee received a recent increase in salary
(d) Communication between the worker and supervisor is empathetic

A

(d) Good treatment outcome is associated with a history of personal empathetic communication

between the supervisor and injured employee. No data show improved outcomes in injured workers

who have recently received a raise or have worked for the employer more than 5 years. Poor

outcomes have been found in injured workers who have received a poor evaluation within the 6

months preceding the injury. The level of position within the workplace is irrelevant in a worker’s

compensation outcome.

133
Q

What work-place situation is the most frequent cause of low back pain in workers?

(a) Jobs that result in falls at work
(b) Jobs that require standing for more than 4 hours
(c) Jobs that requires lifting and material handling
(d) Jobs that require sitting more than 2 hours

A

(c) Jobs that require lifting and material handling place the worker at increased risk for low back
injury. Lifting frequency, load movement, trunk twisting and trunk sagittal angle predict medium

and high-risk occupational low back pain. No risk correlation has been found for length of time

sitting or standing. Although falls in the work place put the worker at risk for low back injury, the repetitive motion involved in lifting and material handling is thought to cause low back pain more frequently.

134
Q

A 14-year-old soccer player seen on the day of injury is unable to bear weight on her right foot. On

examination, she has significant swelling and mild ecchymosis laterally. The patient is tender over

the lateral side of her ankle. X-rays are negative. Your initial plan of management should include

(a) a walking cast for 4–6 weeks.
(b) nonsteroidal anti-inflammatory drugs and contrast baths for 48 hours.
(c) an air stirrup for 2 weeks with weight bearing as tolerated.
(d) an ankle brace and non-weight bearing for 3–4 weeks.

A

(c) This patient has suffered a fairly significant ankle inversion sprain. The lateral collateral ligaments

are injured 85% of the time with this type of injury. This patient should be placed in an air stirrup

for 2 weeks, weight bearing as tolerated, using crutches as needed. A cast or cast boot is acceptable

for 2 weeks but usually is not required. Contrast baths are not indicated before 48 hours following

injury.

135
Q

A functional capacity evaluation is required when

(a) the injured worker is ready to return to his/her job.
(b) the worker’s ability to perform work-related activities must be assessed.
(c) an injured worker’s case remains open more than 7 weeks.
(d) the company physician must determine whether the worker is injured.

A

(b) A functional capacity evaluation (FCE) is an assessment of a worker’s ability to perform workrelated
activities. A functional capacity examination can be used to determine if a worker might

benefit from work hardening or work conditioning, to determine whether a worker can return to

his/her job, to determine if work restrictions are recommended or if job modifications are needed,

and to document the worker’s activity capability. No absolute time line exists. An FCE can be used

in the subacute, or maintenance phase of treatment. An FCE does not determine validity of injury

but can reveal the effort a person expends to perform a task.

136
Q

Which muscle does NOT depress the scapula?

(a) Serratus anterior
(b) Rhomboid major
(c) Latissimus dorsi
(d) Pectoralis minor

A

(b) The rhomboid major elevates, retracts and causes medial (downward) rotation of the scapula, but it

does not depress the scapula. All the other muscles depress the scapula

137
Q

Which of the following may be associated with a subacromial corticosteroid injection?

(a) Dermal keratinification
(b) Localized osteopenia
(c) Tendon rupture
(d) Dermal hyperpigmentation

A

(c) Intra-articular corticosteroid injections have enough systemic absorption that suppression of the

adrenal hypopituitary axis may be seen with repeated injections. More localized deleterious effects

include skin depigmentation, soft tissue atrophy, steroid arthropathy, postinjection flare, and tendon

rupture.

138
Q

How does work conditioning differ from work hardening? Work conditioning is

(a) a maintenance exercise program.
(b) activities simulating the worker’s tasks.
(c) training the worker for a specific job.
(d) aerobic training that may not be job specific.

A

(d) Work conditioning is the physical conditioning portion of work hardening. It has been referred to as

an aerobic training program for patients with less complex and more chronic conditions. Often

these patients are not being retrained to return to a specific job. In a work hardening program the

injured worker is gradually strengthened and reconditioned to the functional capacity level required

to perform a given job. Work hardening has also been referred to as work simulation, work

readiness training, and work rehabilitation.

139
Q

. A 45-year-old man presents to his physician with complaints of posterior heel pain for several weeks.

The pain is worse when he first begins to ambulate after resting. He has a noticeable limp. He denies

any recent trauma. On examination, he is exquisitely tender along the medial and lateral aspects of

the calcaneus and along the Achilles tendon at its most distal portion. The area is mildly swollen and

warm to touch. What is the most likely diagnosis?

(a) Achilles tendon rupture
(b) Plantar fasciitis
(c) Posterior calcaneal nerve entrapment
(d) Retrocalcaneal bursitis

A

(d) This scenario is most likely retrocalcaneal bursitis. This condition is seen in middle aged to elderly

persons, and may occur with overuse. A limp is common and pain is usually worse with first

activity after rest. Swelling and local tenderness may be seen, and a “pump bump” may be present.

This bump is a prominence created by inflammation associated with shoe wear. The local swelling

is due to inflammation of the retrocalcaneal bursa. An acute Achilles tendon rupture is associated

with a positive Thompson test, which consists of compressing the calf and causing the foot to

plantarflex. If the tendon is ruptured, the foot will not plantarflex. The pain is not located in the

bottom of the foot as would be seen with plantar fasciitis. Posterior calcaneal nerve entrapment

would not cause swelling, nor would it be warm to touch.

140
Q

Which statement regarding Waddell signs is TRUE?

(a) They are a collection of organic signs that identify malingerers.
(b) They include simulation, distraction, overreaction, and regional disturbances.
(c) A finding of 2 or more positive signs is clinically significant.
(d) These signs are seen much more commonly in chronic pain patients.

A

b) The Waddell signs are nonorganic physical signs that are used for patients with low back pain.

They are a simple screen that can be used to help identify patients who may require a more detailed

psychological assessment. Three or more positive signs are a clinically significant finding. The

signs include simulation, distraction, overreaction, regional disturbances and either superficial or

nonanatomic tenderness

141
Q

What is the rationale for bracing for low back pain in the injured worker?

(a) It has been shown to prevent further low back pain injury.
(b) It prevents further injury and should be used in all injuries occurring at work.
(c) It has been shown to improve lifting capabilities.
(d) It can provide proprioceptive feedback to reinforce proper mechanics.

A

(d) Lumbar supports do not prophylactically prevent low back injuries or prevent recurrence of low

back pain. One study showed a decrease in lost time at work when lumbar supports were utilized

with a back injury educational program. No improvement in lifting capacity has been found with

the use of bracing. Workers who use lumbar supports use them as a proprioceptive reminder to use

proper mechanics with lifting and other work-related activities

142
Q

Regarding anterior shoulder dislocation, which statement is the most correct?

(a) The most common neural injury is a lower trunk brachial plexus injury.
(b) Return to full contact sports is allowed only after isokinetic values are 100% of normal.
(c) To prevent capsular adhesions, Codman exercises are allowed starting 48 hours postinjury.
(d) Initial rehabilitation (1–3 weeks) should avoid active external rotation past 45°.

A

(d) Following anterior shoulder dislocation, the most common neural injury is damage to the axillary
nerve. Gentle pendular exercises (Codman exercises) are not allowed until weeks 2–3 postinjury.

Early postreduction rehabilitation should avoid active external rotation past 45° for at least 3 weeks

and usually up to 6 weeks. External rotation with abduction is not allowed until 8 weeks postinjury.

143
Q

What information does a functional capacity evaluation provide the treating physician?

(a) It indicates the worker’s ability to return to full duty or return to work with restrictions.
(b) It shows the worker’s maximum oxygen uptake while simulating work related activities.
(c) It provides standardized test results that can be compared with national normative data.
(d) It differentiates discriminates between psychiatric illness and behavioral symptom

magnification.

A

(a) The information provided by functional capacity evaluations (FCE) includes the worker’s capability

to return to full or restricted duty, the need for work hardening, or the need for work conditioning.

It can also define discrepancies between subjective complaints and objective findings. The FCE can

also help determine if the worker has reached maximal medical improvement. Although general

aerobic condition may be assessed, the FCE does not include formal aerobic capacity testing. The

psychological assessment is important during the FCE but formal psychiatric diagnosis is not made

during an FCE. Unfortunately, functional capacity evaluations are not standardized and cannot be

compared. Such standardization has been proposed but currently does not exist.

144
Q
  1. What is the most common level of occult spine fracture after trauma that is missed by plain

radiographs?

(a) C7/T1
(b) T5/T6
(c) T12/L1
(d) L4/L5

A

(a) Occult cervical fractures are most often seen at the C1 and C7 levels. By adding computed

tomography (CT) scanning to the evaluation of trauma patients, a significant number of occult

cervical fractures can be diagnosed. Of spinal fractures, 5%–30% are multiple and may appear at

noncontiguous levels. Thus, radiographic evaluation of the entire spinal axis is necessary whenever

injury at 1 region of the spine is detected.

145
Q

Cumulative trauma disorders

(a) develop as a result of repetitive macrotrauma.
(b) are most common in workers who work at a keyboard.
(c) occur after ergonomic recommendations have been implemented.
(d) result from repetitive microtrauma in the setting of poor ergonomics.

A

(d) Cumulative trauma disorders (CTDs) develop as a result of repetitive microtrauma to tissue in the

setting of poor ergonomics. Any worker who performs a task requiring repetitive loading or motion

is at risk. This includes keyboard operators but is not exclusive to them. The most common

occupational hazard has not been determined. Often, implementing ergonomic recommendations

can help reduce CTDs.

146
Q

A 45-year-old woman with diabetes complains of shoulder pain and stiffness after a minor fall. On

examination you find severely restricted range of motion in all planes. There is no warmth, swelling,

tenderness to palpation, or deformity. Your treatment plan includes

(a) subacromial corticosteroid injection.
(b) an arm sling.
(c) McConnell taping.
(d) Codman exercises.

A

D

The Codman pendulum exercises are recommended to treat adhesive capsulitis (frozen shoulder).

147
Q

A 38-year-old laborer presents with shoulder pain after falling on to the tip of his left shoulder. He

felt immediate pain in the upper part of his shoulder, but no numbness or tingling in his arm. On

examination, he is noted to have a deformity on the superior aspect of his shoulder. He has pain with

horizontal adduction of his left arm across his chest and is having difficulty lifting his left arm. His

passive range of motion is good. Which x-rays with the patient holding and not holding weight(s)

should you order?

(a) A left shoulder anteroposterior radiograph
(b) A scapular Y view radiograph of the left shoulder
(c) Bilateral anteroposterior shoulder radiographs
(d) A left shoulder axillary view radiograph

A

(c) The first step would be to make sure the patient is neurovascularly intact. The next step would be to

obtain bilateral shoulder films with the patient holding and not holding 10-pound weights to

accentuate any laxity present. Bilateral films are necessary as the patient may have lax ligaments in

both shoulders. A scapular Y view is used for shoulder dislocations. An axillary radiograph is used

for shoulder dislocations and will also evaluate the glenoid.

148
Q
  1. On the stand in a civil litigation case, a physiatrist asked about an event causing an injury should
    (a) decline to answer when the worker has had no objective testing that proves a diagnosis.
    (b) answer probable if he/she believes there is more than a 50% chance the event caused the
    injury.
    (c) answer probable if he/she believes there is less than a 50% chance the event caused the injury.
    (d) decline to answer if he/she is not the treating physician.
A

(b) During civil litigation, the physiatrist may be asked to comment on causality of an injury. The

determination that 1 condition caused another condition should be determined on a “reasonable

degree of medical certainty.” In this setting “probable” indicates that the physician believes there is

a greater than 50% chance that condition A caused condition B. The term “possible” indicates that

the physician believes there is a less than 50% chance that 1 event caused another condition.

149
Q

A 39-year-old mail sorter complains of severe right elbow pain for 6 months. He has tried antiinflammatories

without relief. He denies numbness, but does report weakness in his grip. On

examination, the patient has severe pain with palpation just inferior to the lateral epicondyle. Which

of the following findings will most likely be found on further examination?

(a) Pain will be increased with ulnar deviation of the wrist with resisted flexion.
(b) The patient’s brachioradialis reflex will be significantly diminished or absent.
(c) Resisted wrist extension with a straightened elbow will reproduce the patient’s pain.
(d) Atrophy of the extensor digitorum communis and extensor indicis proprius will be present.

A

(c) This patient has lateral epicondylitis or “tennis elbow,” a condition brought on by repetitive flexionextension or pronation-supination of the forearm. The pain will be increased by resisted wrist

extension with the elbow at 180°. The reflexes will not be affected, nor will atrophy be noted. This

is not a neurologic condition, but a myofascial one.

150
Q

Maximum medical improvement is defined as the time when the worker

(a) has completed 3 months of physical therapy.
(b) is not able to perform his/her job because of pain.
(c) is not likely to achieve further improvement of the injury.
(d) is able to return to his/her previous job.

A

(c) Maximum medical improvement is defined as the time when no improvement is likely to further be

achieved after a work-related injury. This definition is exclusive of the amount of time spent in

physical therapy, the worker’s subjective complaints, or whether or not the worker is able to return

to his/her previous job. A worker may reach maximum medical improvement and continue to have

impairments that prevent return to his/her former job.

151
Q

Which financial barrier impedes return to work?

(a) Partial salary and potential monetary gain through litigation
(b) Lack of a retirement plan at work
(c) Low salary prior to the injury
(d) lack of employer-provided health insurance

A

(a) A worker may have 2 financial disincentives to return to work. First, employees usually receive a

portion of their salary while unable to work, if the injury occurred while the worker was performing

his/her job. Second, the worker may not want to return to work if monetary gain is being sought

through pending litigation.

152
Q

The presence of which factor would give a patient a poor prognosis for home discharge after hip

fracture?

(a) Being 85 years old
(b) Having coronary artery disease
(c) Being of male gender
(d) Having a nonhealing wound

A

(a) Factors associated with permanent institutionalization after hip fractures are age greater than 80

years, lack of involvement by family members, insufficient physical therapy at a skilled nursing

facility, and pre-existing dementia.

153
Q

A 32-year-old male runner presents to your office with foot pain for the last 3 weeks. He reports

severe pain on the bottom of his foot, which is worse with the first few steps in the morning after

getting out of bed. He has no history of trauma and previously ran up to 12 miles daily. His running

has been severely limited since this pain began. What is the most likely diagnosis?

(a) Morton’s neuroma
(b) Plantar fasciitis
(c) Tarsal tunnel syndrome
(d) Stress fracture

A

(b) Plantar fasciitis is classically most painful upon arising first thing in the morning, and is aggravated

by overuse or change in footwear. An S1 radiculopathy often presents with numbness and tingling

and has associated reflex changes and possibly weakness in the plantar flexors. Tarsal tunnel

syndrome is caused by compression of the posterior tibial nerve inferior to the medial malleolus. A

Morton’s neuroma causes plantar pain in the forefoot and is aggravated by wearing tight, restrictive

shoes.

154
Q

Which group would NOT be included on a list of occupations with the largest incidences of low

back injuries that receive workman’s compensation?

(a) Truck drivers
(b) House painters
(c) Machine operators
(d) Nurses

A

(b) Alhough, house painters may be at risk for injury, machine operators, truck drivers, and nurses have

the greatest incidence in compensated low back pain injuries

155
Q

A 38-year-old drywall hanger presents with shoulder pain after falling onto the tip of his shoulder.

He felt immediate pain in the upper part of his shoulder, but no numbness or tingling in his arm. On

examination, he has a visible deformity on the superior aspect of his shoulder. He has pain with

horizontal adduction of his left arm across his chest and is having difficulty lifting his arm. His

passive range of motion is good. The best treatment for this patient would involve

(a) use of an arm sling for at least 4 weeks.
(b) referral to an orthopedic surgeon for surgical repair.
(c) physical therapy for Codman exercises.
(d) corticosteroid injection after 10 days.

A

(b) This patient has a grade 3 acromioclavicular separation. Grade 1 or 2 separations would not have a

visible deformity and would require weighted bilateral shoulder films. A grade 3 separation may

have good results with conservative care, but a young manual laborer should be referred for

surgical repair to ensure good results. Grade 4-6 separations should be surgically repaired. Patients

should only be placed in a sling for a few days until the pain subsides. This will decrease the

possibility of losing shoulder range of motion. The shoulder should be given a few days rest, and

physical therapy referral is not appropriate at this time. A corticosteroid injection is not the

treatment of choice and will not repair the separation.

156
Q
  1. Workers who participate in a cardiovascular training program have been found to
    (a) communicate with their supervisors better.
    (b) be more efficient.
    (c) have better job performance evaluations.
    (d) report fewer sick days
A

(d) Workers who participated in a cardiovascular training program were compared to a control group.

Those in the training program reported 51% fewer sick days than controls despite no change in their

maximum oxygen consumption (VO2max).

157
Q

A 42-year-old jackhammer operator presents with low back and left posterior-lateral thigh pain and

numbness that began at the end of his shift 1 week ago. He has been unable to return to his job since

the onset of pain. His physical examination reveals normal reflexes, strength, and sensory function

upon examination of both lower extremities. His pain is reproduced with forward flexion of the

lumbar spine and left straight leg raise. Your recommendations include

(a) ice, muscle relaxants, x-rays, 10 days of bedrest.
(b) nonsteroidal anti-inflammatory drugs, muscle relaxants, lumbar corset, return to work.
(c) nonsteroidal anti-inflammatory drugs, education in positions of comfort, physical therapy.
(d) narcotic pain medications, lumbar corset, lumbar discography

A

c) Nonsteroidal medications, education in lumbar positions of comfort, and physical therapy constitute

the standard of care in conservative management of lumbar radiculopathy. Ten days of bedrest is no

longer recommended, since the effects of immobilization can further impair recovery. Limited or

relative rest can help relieve repetitive trauma while acute pain management interventions are

underway. A lumbar corset may help with pain in the first few days but immediate return to work

while relying on a lumbar corset does aid in recovery. Jackhammer operators are exposed to a great

deal of vibration, which increases an individual’s risk of disc injury. When a disc injury is

suspected in this population, return to a modified work description avoiding lifting, bending,

twisting, and vibration should be recommended. Lumbar discography should be reserved for

individuals

158
Q

. Lumbar spondylolisthesis is the term for slippage of one vertebral body on the adjacent body

below. All of the following statements are true EXCEPT
(a) It is graded 0-4, by the percentage of slippage of the superior body on the inferior one.
(b) It is caused by a fracture or defect in the pars interarticularis.
(c) A TLSO brace is the best method to stabilize an unstable spondylolisthesis.
(d) A spondylolisthesis may cause neurologic compromise of the cauda equina.

A

(c) Spondylolisthesis in the lumbar spine is a common finding, occurring 70% of the time at L5-S1 and

25% at L4-5. It is caused by a defect or fracture in the pars interarticularis and is graded 0-4 on the

basis of the amount of slippage of one body on the other. It may indeed lead to spinal stenosis and

compromise of the cauda equina. A spinal orthosis will not be effective in stabilizing this defect but

can be useful in reducing lumbar lordosis, decreasing pain, and reducing gravitational forces on the

slippage.

159
Q

A 37-year-old male pipefitter has completed physical therapy you prescribed for a C6 radiculopathy.

He no longer requires pain medication and is independent in his home exercise program. He

complains of some pain and fatigue during physical therapy. His neurologic and strength

examination is normal. Your next recommendation is

(a) a functional capacity evaluation.
(b) return to work without restrictions.
(c) vocational rehabilitation.
(d) exercise program with weights at home.

A

(a) A functional capacity evaluation (FCE) is a comprehensive test with some objective data that tests a

person’s ability to perform work-related tasks. An FCE helps determine what the worker can do at

work on a safe and dependable basis. Testing is usually performed work after the initial

rehabilitation program has been completed.

160
Q

. A 42-year-old amateur tennis player complains of severe right elbow pain for 6 months. He has tried

heat, ice, and compression wrap without relief. He denies numbness, but does report weakness in his

grip, especially with his backhand. On examination, the patient has severe pain with palpation just

inferior to the lateral epicondyle. Which finding will most likely be present on further examination?

(a) Pain will be increased with ulnar deviation of the wrist with resisted flexion.
(b) The patient’s brachioradialis reflex will be significantly diminished or absent.
(c) Resisted wrist extension with a straightened elbow will reproduce the patient’s pain.
(d) An audible click will be heard with active supination of the forearm.

A

c) This patient has lateral epicondylitis or “tennis elbow,” a condition brought on by repetitive flexionextension or pronation-supination of the forearm. The pain will be increased by resisted wrist extension with the elbow at 180°. The reflexes will not be affected, nor will atrophy be noted. This is not a neurologic condition, but a myofascial one. No audible click will be heard. This might occur if the radial head is subluxing, but not in lateral epicondylitis.

161
Q
  1. The most common cause of disability in the United States is
    (a) arthritis.
    (b) carpal tunnel syndrome.
    (c) coronary artery disease.
    (d) stroke.
A

(a) Arthritis and other rheumatic conditions are the leading cause of disability in the United States,

imparting an aggregate cost of about 1.1% of the gross national product.

162
Q

A 29-year-old painter presents to you with 2 days of knee pain and swelling after falling off a ladder

at work. The swelling began immediately after the fall. His neurologic examination is normal, and

peripheral pulses are normal at the knee and ankle. He is unable to fully extend or flex the knee

because of pain and swelling. He is ambulating with an antalgic gait limp. Your recommendations

include ice and

(a) knee immobilizer, crutches, x-rays, return to sedentary work, recheck in 5 days.
(b) crutches, magnetic resonance imaging, referral to an orthopaedic surgeon.
(c) narcotics, physical therapy, recheck in 3 weeks.
(d) nonsteroidal anti-inflammatory drugs, x-rays, return to work.

A

(a) Appropriate management for acute knee injuries include, ice, elevation, non-steroidal antiinflammatory

drugs, protection, weight bearing as tolerated and activity modification. X-rays

initially rule out bony injury. magnetic resonance imaging should be reserved for cases where the

diagnosis is in question or a surgical procedure is planned. A careful examination to fully exclude

ligament or cartilage injury cannot be completed until the effusion has resolved enough to allow for

an appropriate examination. Therefore, in the case of an acute knee injury with effusion, the patient

should be reexamined within a 1- to 2-week interval in order to narrow the diagnosis and progress

treatment.

163
Q

A 21-year-old US Army recruit reports to boot camp. After 5 days of marching, he reports to the

base physiatrist with complaints of severe pain in his left shin. He states his pain began after a 10-

mile run in full gear this morning. The pain has gotten significantly worse over the last 2 hours. He

is now unable to bear weight on his left leg. On examination, his left shin is shiny and edematous.

He has severe pain with palpation and the muscles seem tight. The most appropriate treatment plan

for this patient would be to

(a) obtain an x-ray and a triple phase bone scan.
(b) measure the pressure in his tibialis anterior muscle immediately.
(c) wrap the foreleg with an Ace bandage, applying pressure from distal to proximal.
(d) apply ice and have the patient elevate his leg when he gets back to his barracks.

A

(b) Suspicion of a compartment syndrome should lead the physician to get pressure measurements

immediately, since delays may result in permanent muscle or nerve damage. Usual pressures are

less than 30mmHg. Pressures from 30 to 50mmHg are equivocal, but pressures greater than

50mmHg constitute a surgical emergency. The leg should NOT be elevated, because this will lower

arterial perfusion pressure and will further compromise vascular supply. An external

circumferential force will increase pressure. An x-ray and bone scan are not indicated in this

patient.

164
Q

A 32-year-old welder suffered a brachial plexus injury falling off a scaffold. He is unable to use his

right upper extremity because of severe weakness. According to the World Health Organization

classification system, the patient’s weakness describes his

(a) injury.
(b) impairment.
(c) disability.
(d) handicap

A

(b) Impairment is defined as an alteration of a person’s health status, a deviation from normal in a body

part or any organ system (any loss or abnormality of psychologic, physiologic, or anatomic

structure or function).

165
Q

In which case is a corticosteroid injection contraindicated?

(a) A police officer with plantar fasciitis who will return to his usual street duties 2 days after

treatment

(b) A professional tennis player with acute elbow pain associated with her backhand 1 week

before a tournament

(c) A drywall hanger with chronic shoulder pain who has had 1 prior injection 6 months ago with

good results

(d) A diabetic weightlifter with subacute medial knee pain that is warm to touch compared to the

other leg

A

(d) Patients with diabetes mellitus are at risk for serious infection and for systemic effects of absorbed
corticosteroids. Injection into an infected joint, tendon or bursa is contraindicated. Multiple

injections should not be performed unless clear improvement has been demonstrated. More than 3

steroid injections are rarely indicated. There is a risk of tendon rupture in patients who return to

usual activity too rapidly. Plantar fasciitis and lateral epicondylitis respond well to steroid injection.

166
Q

A 50-year-old insulin-dependent diabetic truck driver with a long-standing history of smoking is

sent to you after experiencing left hand weakness following a long haul across the country. Signs

and symptoms include weakness in hand intrinsics and wrist, with numbness in the ring and little

fingers. Exam demonstrates normal upper extremity reflexes. Motor strength is normal for deltoid,

biceps, triceps, pronator teres, and opponens pollicus. There is moderate weakness of the abductor

digiti minimi. Your diagnosis is

(a) C7 radiculopathy.
(b) carpal tunnel syndrome.
(c) ulnar compression at Guyon’s canal.
(d) ulnar neuropathy at the elbow

A

(d) The patient’s symptoms and the physical exam findings are consistent with an ulnar neuropathy at

the elbow.

167
Q

A 55-year-old secretary presents to you with neck, shoulder, and arm pain. Her pain is worse on

Friday than Monday. She denies any trauma or history of previous similar complaints. Her work

station changed a week before the onset of her symptoms. You advise her that

(a) her job has nothing to do with the pain she is experiencing.
(b) taking a week off will reduce symptoms and allow her to return to work without problems.
(c) nonsteroidal anti-inflammatory medications and ice will resolve her problem.
(d) her work station should be evaluated.

A

(d) Postural changes can produce muscle imbalances that can cause pain syndromes. Changing posture

and re-education of muscles through appropriate strengthening programs is appropriate

management. The ergonomics at her work site have a lot to do with her pain complaints. Time away

from work may temporarily reduce symptoms but will likely have no long-term effect. The use of

pain management interventions such as ice and anti-inflammatory medications is appropriate but

should not be expected to correct the long term problem.

168
Q

A 22-year-old skier reports to your mountain ski clinic the day after falling over her ski pole. She

complains of severe pain in her left thumb along the medial aspect of the metacarpophalangeal joint.

She has minimal swelling and no ecchymosis, but is exquisitely tender along the ulnar collateral

ligament. No mediolateral laxity is found on exam. Optimal treatment for this patient should include
(a) immediate referral to a hand surgeon for a grade III ulnar collateral ligament tear.
(b) immobilization in a thumb spica splint for a period of 2 to 4 weeks.
(c) a corticosteroid injection at the site of pain to relieve the inflammation.
(d) no intervention indicated; she should be allowed to return to ski.

A

(b) This patient most likely has suffered a grade I or II ulnar collateral ligament injury. This injury is

called a “gamekeeper’s or skier’s thumb.” A complete (grade III) tear is diagnosed by a difference

of 15° or more of lateral laxity compared to the uninjured side or an absolute laxity of 35°. This is

not a grade III injury, and will likely heal with nonsurgical treatment consisting of immobilization

in a thumb spica cast or splint for 2 to 4 weeks. This patient may be returned to ski with a thumb

spica or cast in place. A steroid injection is not indicated. Ice may help with the pain and swelling,

but immobilization is required for this patient. Nonsteroidal anti-inflammatory drugs may be given

for pain control.

169
Q

Workers who sustain an injury causing continued impairment cannot return to work if

(a) they are at or over the age of 55 years.
(b) their job cannot be modified to accommodate their impairments.
(c) more than 3 months have elapsed since the injury.
(d) the worker was responsible for the injury.

A

(b) If job requirements cannot be modified, an injured worker may not be able to return to his/her job.

The employee’s age or the length of time that has elapsed since the injury does not by definition

exclude the worker from returning to work. Whether the worker was responsible for the injury does

not exclude him/her from returning to work.

170
Q

A data entry clerk presents with complaints of right wrist and hand pain. She attributes the problem

to prolonged use of the computer mouse. Which intervention should be included in the initial

treatment plan?

(a) Eccentric strengthening of the wrist extensors with hand weights
(b) Biweekly steroid injections to minimize symptoms
(c) Wrist splint to minimize symptoms
(d) Moving the mouse away from the keyboar

A

c) Repetitive injuries from keyboard occupations are well recognized and require active rehabilitation

for restorative function. Eccentric exercises will increase repetitive stress to the injured site and

should not be used in the initial treatment recommendations. The prescription of biweekly steroid

injections is excessive and may promote musculotendon atrophy and susceptibility to further injury.

Moving the mouse further away from the keyboard will likely increase symptoms and dysfunctions.

Wrist splints can be used during the initial treatment program to provide relative rest, reduce

inflammation, and to provide comfort.

171
Q

. Which maneuver is used to identify sacroiliac joint pathology?

(a) Lasegue
(b) Gaenslen
(c) Ober
(d) Thomas

A

(b) The Gaenslen sign is a test to determine sacroiliac pathology. This test is performed by having the

patient lie supine. One buttock is extended over the table’s edge while the other remains on the

table. The ipsilateral leg is allowed to drop below the edge of the table, with the other leg remaining

in a flexed position. Pain in the area of the sacroiliac joint on the side of the extended leg represents

a positive test. The Thomas test is a test to measure hip flexion contracture. The Ober test is used to

identify contracture of the iliotibial band or the tensor fascia lata. The Lasegue sign is the straight

leg test, a nerve root stretch test to identify radicular pain.

172
Q

For the past 2 months, a 75-year-old man has had low back pain that radiates in a posterolateral

distribution down his right leg when he stands or ambulates. The pain is relieved by pushing a cart in

the store. He has a non-focal neurologic examination and an x-ray rules out metastatic disease. Your

initial management plan should include

(a) neurosurgical evaluation.
(b) lumbosacral corset.
(c) epidural steroid injection.
(d) flexion based exercises

A

(d) This patient describes classic neurogenic claudication due to spinal stenosis. He has a non-focal

examination and should be managed conservatively initially with flexion based exercises. Epidural

steroid injections may be warranted if the patient does not respond to initial therapeutic exercise.

173
Q

A 19-year-old male competitive swimmer complains of medial knee pain for the last 6 weeks. He

specializes in the breaststroke, and his current practice regimen consists of sprinting 1 hour per day 4

days per week. On exam, he has no effusion or crepitus. His Lachman, McMurray and pivot shift

tests are all negative, and there is no significant laxity. His Q angle is 24° with his knee extended.

His strength is normal. Which action should be recommended?

(a) Strengthen his quadriceps, concentrating on the last 30° of extension.
(b) Modify his swimming program to 3 hours per day of endurance training 6 days per week.
(c) Quit swimming, as he does not have the knee joint architecture for the sport.
(d) Refer for arthroscopic evaluation for internal derangement.

A

(a) The breaststroke has an associated “whip kick” which puts stress on the medial knee. For proper

propulsion, the knee must be in a significant valgus position during leg extension. This can lead to

overuse of the adductors and the quadriceps and a condition known as “breaststroker’s knee.”

Individuals who have patellar tracking problems or a high Q angle (greater than 18° in men) with

the knee extended will be prone to patellofemoral pain. You should prescribe that this young man

work on stretching and strengthening of his adductors and quadriceps. The quadriceps

strengthening should focus on the last 30° of knee extension, which selectively strengthens the

vastus medialis. This patient has negative tests for internal derangement, such as anterior cruciate

ligament tear or meniscus tear. No laxity was noted, and referral for arthroscopy is not indicated.

174
Q

A 45-year-old woman presents with patellofemoral pain syndrome. Which physical finding is

likely?

(a) Positive anterior drawer
(b) Tight lateral thigh structures that cross the joint
(c) Q angle less than 15°
(d) Medial patellar tilt

A

(b) Anterior knee pain syndrome is associated with atrophy of the quadriceps, a large (more than 15°)

Q angle, tight lateral thigh structures, positive posterior drawer, and lateral patellar tilt.

McConnell J, Fulkerson J. The knee:patellofemoral and soft tissue injuries. In: Zachazewski JE, editor.

175
Q

Which statement is TRUE regarding corticosteroid injection for severe carpal tunnel syndrome?

(a) Axonal injury can be successfully reversed with this procedure.
(b) To recognize nerve injection, do not dilute the corticosteroid with anesthetics.
(c) New pain and numbness lasting for more than 48 hours are normal.
(d) Local tenderness and superficial hematomas are rare and indicate a complication

A

(b) Anesthetics mixed with corticosteroid can mask the pain associated with needle placement into the

nerve and should not be used. The risk of intraneural injection is real, but in experienced hands this

injection is safe. Numbness is anticipated with this injection without use of anesthetics, and helps to

confirm proper placement. Local tenderness and hematomas are common with this injection and do

not represent a complication. Persistent or worsening pain, or swelling lasting more than 48 hours,

are signs of nerve injection or neurotoxic injury. Severe carpal tunnel syndrome with axonal loss is

not reversed with this procedure.

176
Q

What is the major determinant of successful return to work after a work related injury?

(a) Amount of lost time
(b) Type of injury sustained
(c) Type of job
(d) Surgical intervention was required

A

(a) The amount of lost time is a major determinant of return to the work place. Although more

extensive injuries may more easily deter the employee from returning to work, the extent or type of

injury has not been found to be the major determinant in all worker’s compensation cases. Neither

the type of job or the type of treatment required, such as surgery, are major determinants of

successful return to work. Several studies have shown that the longer the worker is out of work

related to the injury, the more unlikely it is that he/she will return to work successfully.

177
Q

A 27-year-old female runner presents to your office with foot pain for the last 3 weeks. She reports

a severe pain on the bottom of her foot which is worse with the first few steps in the morning after

getting out of bed. She has no history of trauma and typically runs up to 5 miles per day. Her

running has been severely limited since this pain began. What is the best initial treatment option

for this patient’s pain?

(a) Walking cast for the affected foot
(b) Complete bed rest for several days
(c) Corticosteroid injection at the involved site monthly for 3 months
(d) Foot orthotic and stretching

A

(d) This patient’s symptoms are most consistent with plantar fasciitis. Classically, this syndrome is

most painful first thing in the morning upon arising and is aggravated by overuse or change in

footwear. Stretching the plantar fascia, often before getting out of bed in the morning, and use of a

heel cup or medial longitudinal arch orthotic are the initial treatments for this condition.

Corticosteroid injection may be indicated at the insertion of the fascia into the plantar aspect of the

calcaneus; however, this is usually not required on a repeated basis. Relative rest from the

aggravating activity may be useful, but bed rest is not indicated. A walking cast would not allow

stretch of the plantar fascia.

178
Q

A nurse working in a rehabilitation hospital is most likely to injure her back while

(a) assisting a patient to perform a sliding-board transfer.
(b) catheterizing a patient.
(c) performing a wheelchair-to-toilet patient transfer.
(d) helping a patient move up in bed.

A

(d) The difficulty in using proper posture and body mechanics and the forces required for pulling

patients up in bed are believed to be the reasons that this task most often causes back pain among

nurses.

179
Q

Functional recovery programs for the injured worker with chronic pain should

(a) be done on an inpatient basis to increase the likelihood of success.
(b) not be started until pain medications have been discontinued.
(c) focus on restoration of function.
(d) focus on reducing the patient’s pain symptoms

A

(c) Functional recovery programs should focus on restoring functional ability, including return to

work

180
Q

Which of the following glucocorticoid preparations has the longest therapeutic action?

(a) Triamcinolone diacetate (Aristocort)
(b) Methylprednisolone acetate (Depomedrol)
(c) Hydrocortisone phosphate (Hydrocortone)
(d) Dexamethasone sodium phosphate (Decadron)

A

(d) The onset and length of symptomatic relief after steroid injection are related to the preparation
used. Dexamethasone sodium phosphate has an intermediate solubility and a fairly long biologic

half-life (36-72 hours). Methylprednisolone and triamcinolone have biologic half-lives of 12-26

hours. Hydrocortisone phosphate is not recommended for intraarticular use.

181
Q

A human resource firm working with a manufacturing company inquires about personal factors and

low back injuries. It is interested in matching individuals with work stations. Which of the

following was found to be most important in predicting injuries?

(a) Age greater than 40
(b) Obesity
(c) Psychologic factors
(d) Poor physical fitness

A

(c) Other than a history of previous back injury, psychologic factors were found to be more important

than physical factors in predicting injuries.

182
Q

A 63-year-old man was in a car accident 3 days ago. Cervical radiographs performed in the

emergency department demonstrate multilevel degenerative disc disease. He is currently

asymptomatic but is concerned about the radiographic findings. You inform him that

(a) this is diagnostic of central spinal stenosis.
(b) further neuroimaging is required.
(c) cervical spine immobilization with a soft collar is recommended.
(d) no evaluation or treatment is necessary until symptoms occur.

A

(d) Cervical spine degenerative changes in males over 60 are commonly seen in asymptomatic

patients.

183
Q

. Use of continuous passive motion after total knee replacement surgery

(a) is associated with increased knee flexion at 6 months.
(b) decreases the risk of joint infection.
(c) facilitates knee flexion within the hospital stay.
(d) obviates deep vein thrombosis prophylaxis.

A

(c) Patients treated with continuous passive motion obtain greater early knee flexion (and thus

experience fewer hospital days and manipulations). These devices might exacerbate flexion

contractures and extension lags. Their use has no effect on rates of infection or deep vein

thrombosis.

184
Q

Which of the following is associated with a good outcome following surgical decompression for

spinal stenosis?

(a) Leg more than back pain
(b) Long duration (more than 4 years) of symptoms
(c) Minimal constriction of the spinal canal
(d) Multilevel spondylosis and degenerative joint disease

A

(a) Prognostic factors associated with good surgical outcome following surgical decompression

include pronounced constriction of the spinal canal (<6mm anteroposterior diameter), prominent

leg rather than back symptoms, symptom duration of less than 4 years, and absence of concomitant

disease affecting walking ability.

185
Q

The most significant factor contributing to upper limb cumulative trauma disorders is

(a) size of the equipment used.
(b) age of the person.
(c) macrotrauma to the area.
(d) repeated forceful exertions.

A

(d) Upper limb cumulative trauma disorders, especially those involving the hand and wrist, most often

result from repetitive forceful exertions.

186
Q

A 33-year-old female homemaker presents with pain in the area of the left “anatomic snuff box” for

the last 2 weeks. She has been knitting a blanket for her pregnant sister, and she reports that the

pain has significantly increased since she began this project. On examination, she has exquisite

tenderness to palpation at the site of the snuff box. There is no erythema, but mild swelling is noted

in the painful area. Which of the following statements is true about the patient’s condition?

(a) The inflamed tendon sheath should be injected with a corticosteroid and her hand placed in a

resting splint.

(b) The tendons most likely to be involved are the abductor pollicis brevis and the extensor

pollicis brevis.

(c) This condition is so severe that it warrants referral to a hand surgeon.
(d) She will have a negative Finkelstein’s test.

A

(a) These symptoms are consistent with stenosing tenosynovitis, commonly referred to as de

Quervain’s tenosynovitis. The patient should have tenderness over the common tendon sheath for

the abductor pollicis longus and the extensor pollicis brevis. Finkelstein’s test is confirmative of

this diagnosis. It consists of flexing the thumb under cupped fingers and flexing the wrist in an

ulnar direction, stretching the thumb tendons. This condition usually responds to conservative

management of steroid injection and placement in a splint. Severe refractory cases may require

surgical release, but not until at least a month of conservative therapy has been tried.

187
Q

A 23-year-old painter is recovering from a rotator cuff injury after falling off a ladder. His

impairment would be influenced by

(a) the physical demands of his job.
(b) how he injured the shoulder.
(c) the treatment required.
(d) the extent of limitation of shoulder range of motion.

A

(d) Impairment ratings are based on physical examination findings such as joint limitations and muscle

atrophy and function loss.

188
Q

39-year-old mail sorter complains of severe right elbow pain for 6 months. He has tried antiinflammatory

drugs without relief. He denies numbness, but does report weakness in his grip. On

examination, the patient has severe pain with palpation just inferior to the lateral epicondyle. The

best plan for treating the patient’s condition will include

(a) medical retirement because this condition is resistant to all forms of therapy.
(b) exercise consisting of wrist extensor strengthening and positioning the elbow at 90( flexion

and also full extension.

(c) corticosteroid injection into the elbow joint.
(d) surgical release of the extensor carpi ulnaris tendon to allow the wrist unresisted radial

deviation.

A

(b) This condition can be treated with the use of ice after activity and ultrasound to the inflamed area

at the lateral epicondyle. Steroid and local anesthetic injection into the tendinous insertion can be

useful for temporary relief. A lateral epicondyle counterforce brace (“tennis elbow” band) can

provide relative rest for the inflamed extensor tendons. Exercise using light weights to strengthen

the wrist extensors is useful. This should be done with the elbow flexed and in full extension. If

surgical release is indicated, lateral extensor release is considered to be the procedure of choice.

189
Q

You are providing testimony in a worker’s compensation case. Of the following tests which, would

provide the most objective evidence for a lumbar radiculopathy?

(a) Straight leg raise
(b) Pin prick sensory examination
(c) Electromyogram
(d) Magnetic resonance imaging of the lumbar spine

A

(c) Of the choices provided, electromyogram/nerve conduction velocity testing would provide the most

objective documentation of radiculopathy. Magnetic resonance imaging is an anatomic test.

Straight leg raising and findings of manual strength of 4/5 provide more subjective information.

190
Q

A 21-year-old US Army recruit reports to boot camp. After 5 days of marching, he reports to the

base physiatrist with complaints of severe pain in his left shin. He states that the pain began after a

10-mile run in full gear that morning. The pain has gotten significantly worse over the last 2 hours.

He is now unable to bear weight on his left leg. On examination, his left shin is shiny and

edematous. He has severe pain with palpation, and the muscles seem tight. You are suspicious that

this man has

(a) a severe muscle strain in the tibialis anterior.
(b) a muscle contusion due to a fall while running.
(c) an anterior leg compartment syndrome.
(d) a stress fracture in the left tibia.

A

(c) This patient has been performing excessive, unaccustomed, intense exercise. He is at risk for a

compartment syndrome, which occurs when perfusion of muscle and nerve tissues decreases to a

level inadequate to sustain the viability of the tissues. The intracompartmental pressure increases

and produces venous obstruction. This in turn increases the intracompartmental pressures even

more, and necrosis of muscle and nerve tissue may ensue in as little as 4-8 hours.

191
Q

A 62-year-old stevedore describes experiencing abrupt-onset shoulder pain after lifting a bale. He

has a positive drop arm test and difficulty with overhead reaching activities. He requests magnetic

resonance imaging (MRI) of his shoulder. You tell him that

(a) a rotator cuff tear is unlikely and MRI is not indicated.
(b) a rotator cuff tear is unlikely and MRI would be a very sensitive and specific test.
(c) a rotator cuff tear is likely and MRI will probably be abnormal.
(d) a rotator cuff tear is likely and MRI will probably be normal.

A

(c) Magnetic resonance imaging can identify a large percentage of tears on the rotator cuff of

asymptomatic persons. These tears may be partial or complete. The overall prevalence of

asymptomatic tears of the rotator cuff in all groups was 34%. Fifty-four percent of persons over

age 60 had a tear of the rotator cuff (28% full thickness, 26% partial thickness).

192
Q

Which of the following statements about rupture of the bicipital tendon is true?

(a) Significant deficit in supination strength is usually associated with this condition.
(b) Rupture of this tendon requires surgical repair to maintain functional strength in the biceps.
(c) The tear occurs most commonly in the short head of the biceps and results in only a cosmetic
deformity.

Tears most often occur in persons with long-standing shoulder impingement pain

A

(d) Rupture of the long head of the biceps most often occurs in persons over 40 years of age with longstanding

shoulder pain due to rotator cuff pathology. It may result in approximately 10% loss of

supination in turning the forearm. It does result in a cosmetic deformity with a bulge in the lower

arm, but most patients regain full range of motion and normal elbow flexion strength with a

conservative therapy program. Usually only heavy laborers under age 40 need surgical intervention

to provide the extra strength for lifting.

193
Q

During an independent medical examination, maximum medical improvement is defined by

(a) the original symptoms having resolved.
(b) expected improvement on functional gains having occurred.
(c) the patient having returned to employment.
(d) a period of 12 months having elapsed since the injury.

A

(b) When expected improvement on functional gains has occurred, a patient has reached maximum

medical improvement.

194
Q

Lumbar intradiscal pressure is lowest when a person

(a) stands erect.
(b) sits.
(c) stands with hips flexed.
(d) lies prone.

A

(d) Intradiscal pressure is lowest when a person lies prone and is much higher when standing, running,

sitting, or bending.